Molecular bio final Flashcards

You may prefer our related Brainscape-certified flashcards:
1
Q
An interaction between the hydrogen atom of an hydroxyl group (-OH) with an oxygen atom of a C=O group would be an example of \_\_\_\_\_\_\_\_\_.
A)	a hydrophobic interaction
B)	an oxidative bond
C)	a hydrogen bond
D)	an ionic bond
E)	a covalent bond
A

hydrogen bond

How well did you know this?
1
Not at all
2
3
4
5
Perfectly
2
Q

Which of the following are two techniques in which cDNAs must be generated by using reverse transcriptase in order to do the technique?
A) Two hybrid analysis and microinjection
B) Gene silencing and Western blot
C) RT PCR and Microarray
D) typical PCR and genomic cloning into plasmid vectors
E) Quantitative Real time PCR and Southern blotting

A

RT PCR and Microarray

How well did you know this?
1
Not at all
2
3
4
5
Perfectly
3
Q

You are conducting a microarray experiment with yeast cells and have labeled the cDNAs generated from cells grown under normal condition (N) with a red dye and the cDNAs generated from the same cell line after the cells were subjected to a stress (S) with a green dye. For gene spot 15, you observe that the color was red, what does that mean?
A) The number of chromosomes per cell with gene 15 was higher under normal conditions (N)
B) The expression of gene 15 did not change.
C) Gene 15 was turned on (had more mRNA from gene 15) after stress (S)
D) Gene 15 was turned on (had more mRNA from gene 15) during grown under normal conditions (N)
E) The number of chromosomes per cell with gene 15 was higher after stress (S)

A

Gene 15 was turned on (had more mRNA from gene 15) during grown under normal conditions

How well did you know this?
1
Not at all
2
3
4
5
Perfectly
4
Q

Which of the following is true about the effect of the presence of cholesterol on the fluidity of membranes?
A) Has no effect on the fluidity of membranes
B) Can have either a positive and negative effect on fluidity depending on its concentration in the membrane and the temperature
C) Always reduces fluidity of membranes
D) Can promote fluidity but only in a membrane contain a high concentration of unsaturated fatty acids
E) Always promotes fluidity of membranes

A

Can have either a positive and negative effect on fluidity depending on its concentration in the membrane and the temperature

How well did you know this?
1
Not at all
2
3
4
5
Perfectly
5
Q
Hybridization or annealing of an oligonucleotide primer or nucleic acid probe to a target nucleic acid is used in all of following techniques EXCEPT:
A)	PCR
B)	DNA Sequencing
C)	Southern blot
D)	Microarray analysis
E)	Flow cytometry with FACS analysis
A

Flow cytometry with FACS analysis

How well did you know this?
1
Not at all
2
3
4
5
Perfectly
6
Q

Where would glycolipids (phospholipids with an oligosaccharide attached) be found in cells?
A) In the inner leaflet with the oligosaccharide sticking inside the cell
B) In the cytosol
C) In the outer leaflet with the oligosaccharide sticking outside the cell
D) Inside organelles
E) In the nuclear membrane

A

In the outer leaflet with the oligosaccharide sticking outside the cell

How well did you know this?
1
Not at all
2
3
4
5
Perfectly
7
Q
The yeast two-hybrid method is used 
A)	To generate transgenic animals
B)	To detect if two proteins interact
C)	To separate proteins based on isoelectric point and size
D)	To clone a gene into a plasmid vector
E)	To amplify a gene in two test tubes
A

to detect if two proteins interact

How well did you know this?
1
Not at all
2
3
4
5
Perfectly
8
Q
Which of the following detects the presence of a specific protein with an antibody after separating the protein by size via gel electrophoresis?
A)	Electroporation
B)	PCR
C)	Northern blot
D)	Southern blot
E)	Western blot
A

Western Blot

How well did you know this?
1
Not at all
2
3
4
5
Perfectly
9
Q
Which of the following is a technique that a scientist could use by itself to determine where a specific protein is localizing inside a cell?
A)	Immunofluorescence
B)	Microarray
C)	Northern blot
D)	RT-PCR
E)	C and D
A

Immunofluorescence

How well did you know this?
1
Not at all
2
3
4
5
Perfectly
10
Q

Which of the following would you expect to observe in the membranes of bacterial cells growing at a 37°C as compared to those bacterial cells growing at 15°C?
A) The percentage of saturated fatty acids (ones without a double bond) would be higher in cells at 37°C
B) The percentage of unsaturated fatty acids (ones with a double bond) would be higher in cells at 37°C
C) The percentage of proteins would be higher in cells at 37°C
D) The percentage of cholesterol would increase
E) C and D

A

The percentage of saturated fatty acids (ones without a double bond) would be higher in cells at 37°C

How well did you know this?
1
Not at all
2
3
4
5
Perfectly
11
Q

Which of the following is included in a plasmid vector to facilitate successful cloning?
A) Gene conferring resistance to antibiotic
B) Restriction endonuclease sites
C) Origin of replication
D) All of the above

A

All of the above

How well did you know this?
1
Not at all
2
3
4
5
Perfectly
12
Q
Which of the following is found in all types of cells (prokaryotic and eukaryotic)?
A)	Pili
B)	Nucleus
C)	Ribosomes
D)	Peroxisomes
E)	Endoplasmic reticulum
A

Ribosomes

How well did you know this?
1
Not at all
2
3
4
5
Perfectly
13
Q
Monosaccharides can be linked or attached to \_\_\_\_\_
A)	lipids
B)	another monosaccharides
C)	proteins
D)	an chain of polysaccharides
E)	All of the above
A

all of the above

How well did you know this?
1
Not at all
2
3
4
5
Perfectly
14
Q
Hydrophobic interactions are important for which of the following?
A)	The solubility of sugars in water
B)	The formation of membranes
C)	The folding of proteins
D)	The base pairing of nucleic acids
E)	B and C
A

E) both the formation of membranes and the folding of membranes

How well did you know this?
1
Not at all
2
3
4
5
Perfectly
15
Q

The nematode worm Caenorhabditis elegans is a good model system because it allowed researchers to
A) identify the genes involved in differentiation of every cell type found in mammals
B) observe the process of fertilization better than had been possible in the past
C) follow the process of how cells growing individually can aggregation to form a simple multicellular organism.
D) follow the developmental lineage of all the cells found in an adult from a single fertilized egg cell microscopically

A

follow the developmental lineage of all the cells found in an adult from a single fertilized egg cell microscopically

How well did you know this?
1
Not at all
2
3
4
5
Perfectly
16
Q
Which of the following is NOT found in the plasma membrane of animal cells?
A)	Sphingolipids
B)	Cholesterol
C)	Proteins
D)	Phospholipids
E)	Triacylglycerols or fats
A

Triacylglycerols or fats

How well did you know this?
1
Not at all
2
3
4
5
Perfectly
17
Q

dideoxy nucleotides (ddNTPs) are ________
A) are added in polyermase chain reactions (PCRs) to allow amplication at high temperatures
B) are used in Sanger DNA sequencing to cause chain or strand termination
C) incorporated into RNA in cells
D) are added to cells for gene silencing via RNA interference
E) incorporated into DNA in cells

A

are used in Sanger DNA sequencing to cause chain or strand termination

How well did you know this?
1
Not at all
2
3
4
5
Perfectly
18
Q

A common feature of Xenopus and Zebrafish model systems is ______
A) Short generation time of 2 hours
B) Lineage of every cell in adult in known
C) Easy to grow on defined media
D) Contain chloroplasts
E) Easy to observe development of embryo in microscope

A

E) Easy to observe development of embryo in microscope

How well did you know this?
1
Not at all
2
3
4
5
Perfectly
19
Q
The origin of cellular life is now hypothesized to begin with \_\_\_\_\_\_\_\_\_
A)	DNA inside a carbohydrate layer
B)	RNA inside a protein shell
C)	DNA inside a protein shell
D)	DNA inside a phospholipid membrane
E)	RNA inside a phospholipid membrane
A

RNA inside a phospholipid membrane

How well did you know this?
1
Not at all
2
3
4
5
Perfectly
20
Q
Which type of microscopy has the highest magnification and resolution and is the type scientist use to examine the structure of subcellular organelles and macromolecules?
A)	Scanning electron microscopy
B)	Transmission electron microscopy
C)	Bright-field microscopy
D)	Fluorescence microscopy
E)	Confocal microscopy
A

Transmission electron microscopy

How well did you know this?
1
Not at all
2
3
4
5
Perfectly
21
Q

Flow cytometry and FACS (fluorescent activated cell sorting) is used to ___________
A) To isolate a particular cell component such as the Golgi from a cell lysate by fractionation
B) To detect the presence and level of a particular protein in a lysate generated from cells
C) To isolate cells with a particular set of properties such as presence of the surface marker CD14 from a mixed population of cells
D) To detect the presence of a particular gene in a cell sample
E) To convert RNA to double stranded DNA

A

To isolate cells with a particular set of properties such as presence of the surface marker CD14 from a mixed population of cells

How well did you know this?
1
Not at all
2
3
4
5
Perfectly
22
Q

Which of the following is NOT true about DNA?
A) Usually double stranded
B) Has two antiparallel strands
C) Can fold into an unique 3D shape and act as a catalytic enzyme
D) Has a different sugar in its nucleotides from the one found in RNA nucleotides
E) Has A, C, G and T has bases

A

Can fold into an unique 3D shape and act as a catalytic enzyme

How well did you know this?
1
Not at all
2
3
4
5
Perfectly
23
Q

Which of the following is a property of life that scientists agree viruses can fulfill?
A) Ability to maintain and repair their structure
B) Ability to evolve
C) Ability to transport material in and out
D) Metabolic activity - capture and use energy
E) All of the above

A

Ability to evolve

How well did you know this?
1
Not at all
2
3
4
5
Perfectly
24
Q

To clone a DNA fragment containing a gene into a plasmid vector, one usually does which of the following first?
A) Do density gradient centrifugation of the DNA fragment to separate it from cell components
B) Look at the DNA with a microscope
C) Sequence the gene
D) Hybridize the gene to a microarray
E) Cut both a DNA fragment containing the gene and the plasmid vector with the same restriction endonuclease

A

Cut both a DNA fragment containing the gene and the plasmid vector with the same restriction endonuclease

How well did you know this?
1
Not at all
2
3
4
5
Perfectly
25
Q
Which of the following is a structure that was previously thought to NOT be present in prokaryotic cells but now has been observed in some prokaryotic cells?
A)	Nucleus
B)	Vacuoles
C)	Mitochondria
D)	Cell wall
E)	Actin cytoskeleton filaments
A

Actin cytoskeleton filaments

How well did you know this?
1
Not at all
2
3
4
5
Perfectly
26
Q

Describe the steps a scientist could go through to make a transgenic mouse which a mutation in both of copies of Gene Z.

A

One would make a mutated version of gene Z that would eliminate a portion of it coding region and add a marker. Then one would alter the embryo by either microinjection of eggs or embryonic stem cells. For microinjection, the mutated gene is injected into a pronuclei of a fertilized mouse egg cells and then the altered egg cell is implanted into a foster mother. Offspring are screened for having the mutation. For embryonoic stem, the mutated DNA is introduced into embryonic stem cells growing in culture, a mutated cells is isolated and then it is reintroducted into a blastocyst. The altered blastocyst is then introduced into a foster mother and the offspring will be chimera with normal and mutated cells and will be breed to find one that produces an offspring with a mutated gene Z. In both cases, the initial offspring with a mutated Z will only have one version of gene A mutated and will need to be breed to get offspring with both versions of gene Z mutated.

How well did you know this?
1
Not at all
2
3
4
5
Perfectly
27
Q

Describe the basic steps of a centrifugation technique that one could use to isolate or purify rough ER from the rest of the cell material found in a cell lysate

A

The cells would need be lysed to generate a cell lysate and the one could do either differential centriguation or density gradient centrifugation. For
For Differential centrifugation technique and the following
a. Do a low speed centrifugation
b. spin supernatant at a speed that will remove all more dense cell components
c. Then spin supernatant at speed that will pellet the rough ER. After spin discard supernatant and resuspend pellet containing primarily rough ER.
For Density gradient centrifugation- separates based on buoyancy
1. Load cell lysate onto top of centrifuge tube containing a solution of a dense molecule such as sucrose
2. Spin for many hours
3. Collect fractions from tube going from top or bottom
4. Use fraction that contains rough ER

How well did you know this?
1
Not at all
2
3
4
5
Perfectly
28
Q

Describe two methods that a scientist could use to eliminate the expression or activity of the mouse UNO1 protein produced from the gene uno1 in order to try to determine the function of the UNO1 protein. The uno1 gene is expressed in a mouse cell culture line LilRed1

A

Random mutagenesis of the whole genome of Lilred1 cells and screening for mutants with altered property
Site-directed mutagenesis of a few nucleotides in a clone of the gene to eliminate expression or alter the protein
Total inactivation or knock out of the gene
Gene silencing by using RNA interference or antisense RNA
Make a transgenic mouse strain with a knock out mutation

How well did you know this?
1
Not at all
2
3
4
5
Perfectly
29
Q
The fibrous proteins underlying the inner nuclear membrane are \_\_\_\_\_\_\_?
A)	histones
B)	rans
C)	lamins
D)	importins
E)	nucleosomes
A

lamins

How well did you know this?
1
Not at all
2
3
4
5
Perfectly
30
Q

Which of the following is NOT involved in the transport of proteins into the nucleus?
A) An importin
B) Lamin
C) Ran GEF
D) Sequence on near the amino terminus of proteins
E) Ran

A

lamin

How well did you know this?
1
Not at all
2
3
4
5
Perfectly
31
Q

Two genes associated with inherited breast cancer (BRCA1 and BRCA2) encode proteins that are involved in homologous recombination. Homologous recombination is used to repair ______.
A) uracil base in DNA due to deamination of cytosine.
B) apurinic (AP) sites (no base attached to sugar in DNA).
C) double-strand breaks.
D) O6 methylation (alkylation) of Guanine bases in DNA.
E) pyrimidine dimers.

A

double stranded breaks

How well did you know this?
1
Not at all
2
3
4
5
Perfectly
32
Q

Which of the following describes the relationship that has been observed in sequenced genomes between genome size and the percent protein coding?
A) A positive relationship was observed with a strong correlation (r2 > 0.9)
B) A positive relationship was observed with a weak correlation (r2 between 0.7-0.9)
C) No correlation was observed
D) A negative or inverse relationship was observed with a strong correlation (r2 > 0.9)

A

A negative or inverse relationship was observed with a strong correlation (r2 > 0.9)

How well did you know this?
1
Not at all
2
3
4
5
Perfectly
33
Q
Which of the following would be the optimal alignment between the two sequences
A)	AATCGGA
ACTCG-A
B)	AATCGGA
-ACTCGA
C)	AA-TCGGA
-ACTCG-A
D)	A-ATCGGA
-ACTCG-A
A

A). AATCGGA

ACTCG-A

How well did you know this?
1
Not at all
2
3
4
5
Perfectly
34
Q

30 nm fibers are _____
A) the fibers that attach to the centromere
B) only observed in condensed chromosomes undergoing segregation during mitosis and meiosis
C) the fibrous network on the inside of the nuclear envelope
D) composed of a string of nucleosomes arranged in a helix or spiral

A

composed of a string of nucleosomes arranged in a helix or spiral

How well did you know this?
1
Not at all
2
3
4
5
Perfectly
35
Q

How many telomeres, centromeres and origins of replication will be found on a single eukaryotic chromosome?
A) Many telomeres, many centromeres and one origin of replication
B) No telomere, one centromere and one origin of replication
C) Two telomeres, two centromeres and many origins of replication
D) One telomere, one centromere and one origin of replication
E) Two telomeres one centromere and many origins of replication

A

Two telomeres one centromere and many origins of replication

How well did you know this?
1
Not at all
2
3
4
5
Perfectly
36
Q

A pseudogene is a
A) gene that arose through gene duplication, but by acquiring mutations became nonfunctional.
B) gene that evolved by gene duplication and the acquisition of mutations to yield a gene product that has a slightly different function from that of the original gene product.
C) second copy of a gene that functions when the original copy becomes damaged.
D) gene that is unrelated in sequence to another gene but has the same function.

A

gene that arose through gene duplication, but by acquiring mutations became nonfunctional.

How well did you know this?
1
Not at all
2
3
4
5
Perfectly
37
Q

Which of the following statements concerning elongation of DNA at the replication fork is false?
A) Both strands are synthesized continuously at the replication fork.
B) The Okazaki fragments are joined by the action of DNA ligase.
C) The leading strand is synthesized continuously in the direction of replication fork movement.
D) The lagging strand is synthesized in 1 kb segments called Okazaki fragments backward from the overall direction of replication.

A

Both strands are synthesized continuously at the replication fork.

How well did you know this?
1
Not at all
2
3
4
5
Perfectly
38
Q
You are doing a sequence alignment.  Which of the places the following alignment outcomes in the correct order from most positive value to most negative value?
A)	match, mismatch, gap
B)	match, gap, mismatch
C)	gap, match, mismatch
D)	mismatch, match, gap
A

match, mismatch, gap

How well did you know this?
1
Not at all
2
3
4
5
Perfectly
39
Q

What is the role of the clamp-loading protein in replication?
A) Links together the Okazaki fragments
B) Unwinds the two strands of DNA
C) Loads the protein that rewinds the strands after replication
D) Helps to coordinate the replication of both strands by binding to the DNA polymerase acting on each strand and the helicase at the front of the replication fork
E) Synthesizes short RNA segments on the leading strand

A

Helps to coordinate the replication of both strands by binding to the DNA polymerase acting on each strand and the helicase at the front of the replication fork

How well did you know this?
1
Not at all
2
3
4
5
Perfectly
40
Q

Which of the following is the mechanism by which most proteins transit the nuclear envelope?
A) Active transport through a nuclear pore that is powered by cleavage of a GTP by a G protein involved in the transport process
B) Passive diffusion through the nuclear membrane phospholipid bilayer
C) Active transport through a nuclear pore that is powered by phosphorylation of the protein being transported
D) Active transport through a nuclear pore that is powered by the cleavage of glucose by the nuclear pore complex
E) Passive diffusion through a nuclear pore

A

Active transport through a nuclear pore that is powered by cleavage of a GTP by a G protein involved in the transport process

How well did you know this?
1
Not at all
2
3
4
5
Perfectly
41
Q

Export of RNAs from the nucleus occurs primarily by
A) cotranscriptional insertion through nuclear envelope membrane protein pores.
B) release from the nucleus when it breaks down at mitosis.
C) passive diffusion through nuclear pore complexes.
D) selective transport through nuclear pore complexes in association with proteins.

A

selective transport through nuclear pore complexes in association with proteins.

How well did you know this?
1
Not at all
2
3
4
5
Perfectly
42
Q
You are analyzing the sequence of a chromosome from the genome of the chimpanzee and have found a region on the chromosome where there is 300 copies of same 10 base long sequence arranged in tandem i.e. ...AGTTCAATCGAGTTCAATCG...  .  What type of genomic element have you found?
A)	A DNA transposon
B)	A simple sequence repeat
C)	A gene family
D)	A SINE
E)	A LINE
A

a simple sequence repeat

How well did you know this?
1
Not at all
2
3
4
5
Perfectly
43
Q

Which of the following is NOT true about chromatin?
A) Uncondensed areas are usually transcriptionally inactive
B) DNA in 30 nm fibers attached to scaffolding proteins to form loops
C) DNA is wrapped around histone octomer
D) Some regions are highly condensed (heterochromatic) and other regions are less condensed
E) Non-histone proteins are a major component

A

Uncondensed areas are usually transcriptionally inactive

How well did you know this?
1
Not at all
2
3
4
5
Perfectly
44
Q

Humans with the inherited disease xeroderma pigmentosum (XP) rapidly develop skin cancer if exposed to ultraviolet light. These individuals have a genetic mutation within one of the gene that encodes a protein involved in nucleotide-excision repair. This suggests ______
A) that the principal means that humans use to repair double strands breaks is nucleotide-excision repair
B) that humans contain a photolysase that can do photoreactivation
C) that replication requires the enzymes involved in nucleotide-excision repair
D) that the principal means that humans use to repair pyrimidine dimers is nucleotide-excision repair
E) that the principal means that humans use to repair alkylated bases is nucleotide-excision repair

A

that the principal means that humans use to repair pyrimidine dimers is nucleotide-excision repair

How well did you know this?
1
Not at all
2
3
4
5
Perfectly
45
Q

Which of the following is a difference between the genome of the bacterium E. coli and the genome of the humans?
A) E. coli has more repetitive DNA sequence elements
B) E. coli has more genes
C) The size of the E. coli genome is larger
D) E. coli has introns
E) E. coli has a higher percentage of sequences that code for proteins

A

E. coli has introns

How well did you know this?
1
Not at all
2
3
4
5
Perfectly
46
Q

Blast at the National Center for Biotechnology Institute (NCBI) can be used ____.
A) to look for stored nucleotides sequences that are similar to a query nucleotide sequence
B) to look for stored protein sequences that are similar to a query amino acid protein sequence
C) to look for information about a query gene name
D) to look for journal articles about a query topic
E) A and B

A

E) both

  • to look for stored nucleotides sequences that are similar to a query nucleotide sequence
  • to look for stored protein sequences that are similar to a query amino acid protein sequence
How well did you know this?
1
Not at all
2
3
4
5
Perfectly
47
Q

To sequence the human genome, the Celera Genomics team led by Craig Venter
A) used sequence-cloned DNA and then mapped it by FISH.
B) used the shotgun approach to sequence fragments and assemble them in order using overlaps between the sequences.
C) used BAC clones as substrates for sequencing.
D) All of the above

A

used the shotgun approach to sequence fragments and assemble them in order using overlaps between the sequences.

How well did you know this?
1
Not at all
2
3
4
5
Perfectly
48
Q
The nuclear localization signal is recognized by and binds to which protein in the process of nuclear protein import?
A)	Importin
B)	Exportin
C)	The outer fibril protein
D)	Ran
A

Importin

How well did you know this?
1
Not at all
2
3
4
5
Perfectly
49
Q

A gene family is a
A) family of individuals with the same gene.
B) set of slightly different genes present as one copy each in a set of individuals.
C) family of individuals in which each has a slightly different sequence of the same gene.
D) set of related but slightly different genes present in multiple copies in one individual.

A

set of related but slightly different genes present in multiple copies in one individual.

How well did you know this?
1
Not at all
2
3
4
5
Perfectly
50
Q

A major component of ribosomes is rRNAs. The rRNAs must be synthesized, processed and then are assembled with ribosomal proteins to form the two major subunits of ribosomes. Where does each of these steps occur within an eukaryotic cell?
A) All three occur in the nucleolus region within the nucleus
B) Synthesis of rRNA occurs in the nucleus while processing and assembly occur in the cytoplasm
C) Synthesis of rRNA occurs in the nucleus while processing and assembly occur in the mitochondria
D) Synthesis and processing of rRNA occurs in the nucleolus region within the nucleus while assembly occurs in the endoplasmic reticulum
E) All three steps occur in the cytoplasm

A

All three occur in the nucleolus region within the nucleus

How well did you know this?
1
Not at all
2
3
4
5
Perfectly
51
Q

Which of the following is a similarity between LINEs, SINEs and retroviral elements?
A) All are a minor component of the genome of humans (represent only 1% of the genome)
B) All have long terminal repeats (LTRs) on each end
C) All are usually found in multiple tandem repeats or multiple copies in a location in the chromosome
D) All are thought to have been generated from RNA by reverse transcriptase

A

All are thought to have been generated from RNA by reverse transcriptase

How well did you know this?
1
Not at all
2
3
4
5
Perfectly
52
Q

Bioinformatics is a field of science seeking to
A) develop algorithms that will convert information into biological polymers for storage
B) develop algorithms to analyze and extract useful information from biological polymer sequences in order to understand the function of molecules in cells
C) develop computers that will read biological polymers directly
D) develop computers to act like living beings

A

develop algorithms to analyze and extract useful information from biological polymer sequences in order to understand the function of molecules in cells

How well did you know this?
1
Not at all
2
3
4
5
Perfectly
53
Q

A centromere is defined as a region of chromosome that
A) is very highly condensed and resembles the chromatin of cells undergoing mitosis.
B) plays a critical role in ensuring the correct distribution of duplicated chromosomes to daughter cells during mitosis.
C) is relatively decondensed and distributed throughout the nucleus.
D) is located at the ends of the chromosomes and plays a critical role in chromosome replication and maintenance.

A

plays a critical role in ensuring the correct distribution of duplicated chromosomes to daughter cells during mitosis.

How well did you know this?
1
Not at all
2
3
4
5
Perfectly
54
Q
Which of the following was discovered to account for only 1-1.5% of the human genome?
A)	Pseudogenes
B)	Transposable elements
C)	Protein coding regions
D)	Introns
E)	Simple-sequence repeats
A

Protein coding regions

How well did you know this?
1
Not at all
2
3
4
5
Perfectly
55
Q

What are two bad things could happen to a chromosome if it did not have telomeres?

A
  1. The chromosome could lose sequences of each end at each round of replication.
  2. The linear ends of be degraded by nucleases in the cell without the proteins that bind to the telomeres
  3. The DNA ends might be treated like a double strand break and linked to other chromosomes
How well did you know this?
1
Not at all
2
3
4
5
Perfectly
56
Q

Describe four different mechanisms that could cause an alteration of DNA that could produce a mutation in a DNA sequence.

A

Errors in replication when polymerase incporates the incorrect nucleotide
Spontaneous or chemically induced lose of a base to create a AP site
Lose of amino group on a base which alters it base pairing.
Addition of a alkyl (methyl or ethyl) group to a base that alter base pairing
ultraviolet radiation causing a pyrimidine (thymine) dimer
Ionizing Radiation causing double strand break which are fused with a lose of some bases
Transposition of transposons into a new site
chromosomal rearrangement between two different chromosomes

How well did you know this?
1
Not at all
2
3
4
5
Perfectly
57
Q

Describe two mechanisms that a cell could use to repair a pyrimidine dimer in its DNA.

A
  1. Photoreaction by a photolysase enzyme cleaving the bonds between the two adjacent pyrimidine bases
  2. Nucleotide excision repair where the DNA strand with the dimer is cleaved o n both sides of the dimer and then replaced with new strands
  3. Transcription coupled repair, if RNA polymerase stalls at the dimer, repair enzymes will be recruited and preform nucleotide excision repair
  4. Translesion repair - if encountered while strand being replicated, a different polymerase can come in and go past the dimers
How well did you know this?
1
Not at all
2
3
4
5
Perfectly
58
Q
  1. Describe the composition and structure of a telomere.
A

Telomeres are the sequences located on the end of eukaryotic chromosomes and consists of many repeats (80-200) of a short sequence element (5-7 bp). At the very end, a short section is only single stranded and can anneal with an interior section to form a loop. Specific proteins bind to the telomere DNA.

How well did you know this?
1
Not at all
2
3
4
5
Perfectly
59
Q

Which of the following is NOT true about the specific transcription factors that bind to enhancers in eukaryotes?
A) Most act as repressors of transcription
B) Many are dimers
C) May cause DNA to loop out or bend
D) Most interact with RNA polymerase through a coactivator complex
E) Most work cooperatively with other transcription factors to regulate expression

A

Most act as repressors of transcription

How well did you know this?
1
Not at all
2
3
4
5
Perfectly
60
Q

How are proteins transported into the mitochondria?
A) Posttranslationally after the entire protein is synthesized by the ribosome mediated by specific
signal sequence
B) Via vesicles from the Golgi
C) Via vesicles from the endoplasmic reticulum
D) Cotranslationally via a signal sequence similar to how proteins are transported into the ER in
animal cells

A

Posttranslationally after the entire protein is synthesized by the ribosome mediated by specific

How well did you know this?
1
Not at all
2
3
4
5
Perfectly
61
Q

Which of the following happens the earliest in the formation of an eukaryotic RNA polymerase II preinitiation complex?
A) The binding of a specific transcription factors to the TATA box
B) The binding of complex of general transcription factors called TFII to the promoter
C) The association of general transcription factors with RNA polymerase II
D) The phosphorylation of the carboxy terminal domain (CTD) of RNA polymerase II
E) The binding of RNA polymerase II to the promoter

A

The binding of complex of general transcription factors called TFII to the promoter

How well did you know this?
1
Not at all
2
3
4
5
Perfectly
62
Q

Which of the following is a usually observed in transcriptionally active regions of chromosomes?
A) The cytosine bases of CG pairs are methylated
B) The presence of many simple sequence repeats
C) The N-terminal tails of histone H3 and H4 are acetylated
D) Cytosine bases have been converted to uracil
E) All of the above

A

The N-terminal tails of histone H3 and H4 are acetylated

How well did you know this?
1
Not at all
2
3
4
5
Perfectly
63
Q

Helix-turn-helix and zinc fingers are examples of
A) Dimerization domains of transcription factors
B) Activation domains of transcription factors
C) Inhibitor domains of repressors
D) DNA methylation domains
E) DNA binding domains of transcription factors

A

DNA binding domains of transcription factors

How well did you know this?
1
Not at all
2
3
4
5
Perfectly
64
Q

How are proteins to be secreted now thought to move through the compartment or sacs of the Golgi apparatus?

A) The proteins are transported in vesicles from one compartment or sac of the Golgi to the next compartment or sac
B) The proteins after entry will diffuse throughout all the lumenal volume of the Golgi sacs and then associate with a vesicles that can form and leave on any face of the Golgi and go to the plasma membrane
C) The proteins enter the trans face and then the compartment it entered matures and advances toward the cis side of the Golgi organelle
D) The proteins enter the cis face and then the compartment it entered matures and advances toward the trans side of the Golgi organelle

A

The proteins enter the cis face and then the compartment it entered matures and advances toward the trans side of the Golgi organelle

How well did you know this?
1
Not at all
2
3
4
5
Perfectly
65
Q

Which of the following is NOT true?
A) Each subunit of the ribosome is composed of at least one RNA and several proteins
B) The peptide being synthesized is attached to the ribosome
C) RNA is the largest component by mass of the ribosome
D) The RNA in the ribosome plays a catalytic role in protein synthesis
E) The ribosome is composed of RNA and protein

A

The peptide being synthesized is attached to the ribosome

How well did you know this?
1
Not at all
2
3
4
5
Perfectly
66
Q

Termination of transcription in the prokaryotic bacterium E. coli occurs by
A) formation of a stem-loop structure in the RNA.
B) binding of Rho protein to the end of the mRNA.
C) binding of a sigma () factor to the end of the mRNA.
D) Either a or b

A

D

How well did you know this?
1
Not at all
2
3
4
5
Perfectly
67
Q

The interaction of specific transcription factors with RNA polymerase through a complex of proteins called mediator is observed _________.
A) in both prokaryotic and eukaryotic cells
B) in neither prokaryotic and eukaryotic cells
C) only in eukaryotic cells
D) only in prokaryotic cells

A

only in eukaryotic cells

How well did you know this?
1
Not at all
2
3
4
5
Perfectly
68
Q

Which of the following is NOT true?
A) The mRNA is capped and polyadenylated before splicing occurs
B) For some genes, different sequences are spliced out of the same pre-mRNA
C) Splicing can be mediated solely by the RNA sequence without the help of proteins
D) Introns are spliced out in the cytoplasm after the mRNA is transported out of the nucleus
E) Splicing of most mRNA in eucaryotes is mediated by a complex of RNA and protein

A

Introns are spliced out in the cytoplasm after the mRNA is transported out of the nucleus

How well did you know this?
1
Not at all
2
3
4
5
Perfectly
69
Q

If an integral membrane protein destined for the plasma membrane is oriented with its amino (N) terminus in the lumen of the ER, where will the amino-terminus of the protein reside when the protein sits in the plasma membrane?
A) The location of the amino terminus can vary because the orientation of some integral membrane
proteins can be flipped in the Golgi
B) The amino terminus will be in the cytoplasm
C) The location of the amino terminus can vary because the orientation of some integral membrane
proteins can be flipped in transport vesicles
D) The amino terminus will be on the outside of the cell

A

The amino terminus will be on the outside of the cell

How well did you know this?
1
Not at all
2
3
4
5
Perfectly
70
Q

Vesicles that carry proteins from the rough ER to the Golgi apparatus bud off as _______ vesicles.
A) COPII-coated B) uncoated
C) clathrin-coated D) COPI-coated

A

COPII-coated

How well did you know this?
1
Not at all
2
3
4
5
Perfectly
71
Q

N-linked oligosaccharides are added in the
A) cis Golgi and modified in the trans Golgi.
B) cis Golgi and modified in the medial Golgi.
C) ER and modified in the Golgi.
D) medial Golgi and modified in the trans Golgi.

A

ER and modified in the Golgi.

How well did you know this?
1
Not at all
2
3
4
5
Perfectly
72
Q

Which of the following is NOT true about enhancers in plant and animal cells?
A) Can function if located downstream of the promoter
B) Can function if orientation of the element is flipped
C) Can function if upstream of the promoter
D) Can only function if located within 1 kB of the promoter

A

Can only function if located within 1 kB of the promoter

How well did you know this?
1
Not at all
2
3
4
5
Perfectly
73
Q
What else occurs in the ER and Golgi besides protein processing and sorting?
A) mRNA processing
B) Endocytosis
C) Nucleic acid synthesis
D) Lipid synthesis and modification
E) Photosynthesis
A

Lipid synthesis and modification

How well did you know this?
1
Not at all
2
3
4
5
Perfectly
74
Q

Which of the following describes the mechanism by which the ribosome determines where to start translation in a prokaryotic bacterial cell?
A) Starts at the first AUG it encounters as it moves 3’ to 5’ from the 3’ poly A tail
B) Starts at an AUG that is about 10 pair upstream (in 5’ direction) from a Shine-Delgarno sequence
C) Starts at an AUG or GUG that is about 10 base pair downstream (3’) of a Shine-Delgarno seqence
D) Starts at a AUG located within a Shine-Delgarno sequence
E) Starts at the first AUG it encounters as it moves 5’ to 3’ from the 5’ cap

A

Starts at an AUG or GUG that is about 10 base pair downstream (3’) of a Shine-Delgarno seqence

How well did you know this?
1
Not at all
2
3
4
5
Perfectly
75
Q

How is a lysosome formed?
A) A vesicle from the plasma membrane that has its clathrin coat removed will become a lysosome
B) Vesicles from the Golgi carrying lysosomal proteins fuse with an endocytic vesicle or
phagosome and the proteins cause the vesicle to mature into a lysosome
C) A vesicle leaving the Golgi will become a lysosome if it has a clathrin coat
D) All vesicles that leave the endosome become a lysosome
E) A vesicle buds off the Golgi as a lysosome

A

Vesicles from the Golgi carrying lysosomal proteins fuse with an endocytic vesicle or
phagosome and the proteins cause the vesicle to mature into a lysosome

How well did you know this?
1
Not at all
2
3
4
5
Perfectly
76
Q

Which of the following is NOT a mechanism by which microRNA can cause RNA interference or gene silencing?
A) Can interfere with the RNA polymerase as it is transcribing a particular RNA
B) Can cause a particular RNA to be cleaved and degraded
C) Can block translation of a particular RNA
D) Can remodel the chromatin of a particular gene to inhibit transciption

A

Can interfere with the RNA polymerase as it is transcribing a particular RNA

How well did you know this?
1
Not at all
2
3
4
5
Perfectly
77
Q

What processing occurs on the 5’ end of mRNA in eukaryotic cells?
A) Addition of a single guanosine nucleotide
B) Addition of many adenosine nucleotides
C) Nothing
D) Addition of multiple guanosine nucleotides
E) Addition a single adenosine nucleotide

A

Addition of a single guanosine nucleotide

How well did you know this?
1
Not at all
2
3
4
5
Perfectly
78
Q

Which of the following types of processing of RNA occurs in prokaryotic cells?
A) Addition of poly A tail to the 3’end of mRNA
B) Capping of the 5’end of mRNA
C) Cutting our of rRNAs and tRNAs from larger precursors
D) Splicing out of introns by a complex of RNA and proteins
E) None of the above

A

Cutting our of rRNAs and tRNAs from larger precursors

How well did you know this?
1
Not at all
2
3
4
5
Perfectly
79
Q

Which of the following most commonly mediates the import of proteins into the ER in mammals?
A) Signal cap on 5’ end of mRNA
B) Signal sequence on the carboxy terminus of the protein
C) Signal sequence in the interior of the protein
D) Signal sequence on 3’ of mRNA
E) Signal sequence on the amino terminus of protein

A

Signal sequence on the amino terminus of protein

How well did you know this?
1
Not at all
2
3
4
5
Perfectly
80
Q

Which of the following is NOT observed in prokaryotic cells?
A) Transcription factors are not required for initiation of transcription to occur
B) Multiple genes or coding regions can be present on same mRNA
C) One type of RNA polymerase mediates production of all types of RNA
D) Introns are spliced out of mRNA
E) Transcription and translation are coupled or can occur same time

A

Introns are spliced out of mRNA

How well did you know this?
1
Not at all
2
3
4
5
Perfectly
81
Q

If an animal cell had a mutation that prevented it from making a t-SNARE that normally resides in the plasma membrane, which of the following would be the most likely to be impaired?
A) The formation of vesicles that would carry secretory vesicles to the plasma membrane
B) The fusion of phagosomes with the Golgi apparatus
C) The formation of clathrin coated vesicles in the Golgi
D) The fusion of vesicles whose target membrane is the plasma membrane
E) The formation of vesicles that would carry endocytic vesicles to the endosome

A

The fusion of vesicles whose target membrane is the plasma membrane

How well did you know this?
1
Not at all
2
3
4
5
Perfectly
82
Q

Which of the following is NOT true about decoding of mRNA during translation?
A) mRNA is read 5’ to 3’
B) All codons that code for a same amino acid are used equally in all organisms
C) More than one codon can code for the same amino acid
D) Three possible stop codons
E) Continuous reading of sets of 3 nucleotides at a time with no overlaps or skips

A

All codons that code for a same amino acid are used equally in all organisms

How well did you know this?
1
Not at all
2
3
4
5
Perfectly
83
Q

Which of the following is NOT true about the carboxy-terminal domain (CTD) of eukaryotic RNA
polymerase II?
A) Domain of RNA polymerase to which specific transcription factors interact to stimulate initiation
B) Consists of a repetitive sequence of amino acids
C) Can be reversible phosphorylated
D) RNA processing enzymes associate with the tail during elongation
E) Plays a role in switching RNA polymerase II from initiation state to elongation state

A

Domain of RNA polymerase to which specific transcription factors interact to stimulate initiation

How well did you know this?
1
Not at all
2
3
4
5
Perfectly
84
Q

G proteins that bind and hydrolyze GTP are directly involved in all of the following EXCEPT
A) Import of proteins into the nucleus
B) Transcription initiation
C) Fusion of vesicles with their target membrane
D) Translation elongation cycle

A

Transcription initiation

How well did you know this?
1
Not at all
2
3
4
5
Perfectly
85
Q

Amino acids are added to the _____ of tRNA by aminoacyl tRNA synthetase to make charged tRNA.
A) the 3’ Thymine nucleotide
B) the 5’ 7-methylguanosine nucleotide cap
C) the anticodon
D) the 3’ Adenine nucleotide
E) the 5’ AUG

A

the 3’ Adenine nucleotide

How well did you know this?
1
Not at all
2
3
4
5
Perfectly
86
Q

The addition of which of the following to a protein can be a signal that leads to the degradation of the modified protein?
A) Multiple kinesin proteins
B) Multiple ubiquitin protein molecules
C) A palmitic acid group
D) A prenyl group
E) A glycosylphosphatidylinositol (GPI) group

A

Multiple ubiquitin protein molecules

How well did you know this?
1
Not at all
2
3
4
5
Perfectly
87
Q
Alanine is coded for by 4 codons. How would you expect that the four codons found in mRNA differ?
A) Differ in third position
B) Differ in first position
C) Differ in all 3 positions
D) Differ in second position
E) Differ in second and third position
A

Differ in third position

How well did you know this?
1
Not at all
2
3
4
5
Perfectly
88
Q

Which of the following lists the events that occur in splicing in the correct order?
A) Formation of lariat in the intron, splicing at the 3’ splice site and cutting at the 5’ splice site
B) Cutting at 3’ splice site, cutting at 5’ splice site and formation of lariat in intron
C) Cutting at the 5’ splice site, formation of lariat in intron and cutting at the 3’ splice site
D) Cutting at 3’ splice site, formation of lariat in intron and cutting at the 5’ splice site

A

Cutting at the 5’ splice site, formation of lariat in intron and cutting at the 3’ splice site

How well did you know this?
1
Not at all
2
3
4
5
Perfectly
89
Q

When the two ribosomal subunits are assembled for the beginning of the initiation of translation, the initiator tRNA ______
A) is located in the A (acceptor) site
B) is not associated with the assembled ribosome
C) is located in the P (peptide) site
D) is located in the E (exit) site.

A

is located in the P (peptide) site

How well did you know this?
1
Not at all
2
3
4
5
Perfectly
90
Q

Proteins can be modified by the addition of other types of organic molecules. List three molecules that can be added to a protein and then tell how it is attached to a protein

A

A unit of 14 sugars can be added via an amino group of an asparagine amino acid (N-linkage) A single sugar can be added via the oxygen of a serine or theorine amino acid (O-linkage)
A myristyl fatty acid can be added to the N-terminal glycine
A prenyl lipid group can be added to a cysteine amino acid near the C-terminus
A palmitic acid fatty acid can be added via a sulfur of cysteine
A glycosylphosphatidylinositol (GPI) can be added to the C-terminus
A phosphate can be added via serine, threonine or tyrosine
Ubiquitin proteins can added

How well did you know this?
1
Not at all
2
3
4
5
Perfectly
91
Q

Describe a mechanism for how an integral membrane protein gets inserted into the membrane during synthesis?

A
  1. Have signal sequence and start getting pumped into ER and then have a hydrophobic stop transfer sequence which prevents further pumping into the ER and then the protein escape the translocon pore and inserts into the membrane so the C-terminus is located in the cytosol.
  2. May have an internal signal sequence in middle of protein that causes protein to be pumped into translocon and then remainder of protein could be pumped into ER or could remain outside depending or orientation needed
How well did you know this?
1
Not at all
2
3
4
5
Perfectly
92
Q

Give three differences between transcription in prokaryotes and eukaryotes. The differences could be related to polymerases, initiation, mRNA or regulation? For each difference describe the feature or property in each type of cell (3 pts)
Prokaryotes Eukaryotes

A
Prokaryote
One type of RNA pol
RNA pol has 5-6 subunits
no general transcription factors
Promoter consist in -35 and -10 region Regulatory sequences close to the initiation site upstream or downstream
Polycistronic mRNA
Transcription and translation coupled
mRNA not processed (no introns)
tail)

Eukaryotes
4-5 types of RNA pol
RNA pol has 14-17 subunits
Large complex of general txn factors required
TATA box and initiator element
Regulatory sequences can be up to 100 kB away, and either
Monocistronic mRNA (one gene per mRNA) Transcription and translation separate
mRNA processed (capped, introns spliced out, and polyA

How well did you know this?
1
Not at all
2
3
4
5
Perfectly
93
Q

Which of the following is NOT true about oncogenes?
A) Encode a protein that plays a role in stimulating growth
B) Can be created by a chromosomal translocation or rearrangement
C) Can be created by a point mutation
D) Are only seen in cancer cells caused by tumor viruses
E) Are usually an altered form of a gene normal found in an organism

A

Are only seen in cancer cells caused by tumor viruses

How well did you know this?
1
Not at all
2
3
4
5
Perfectly
94
Q

How are embryonic stem cells different from most adult cells?
A) Embryonic stem cells are infected by viruses
B) Embryonic stem cells grow slower
C) Embryonic stem cells are pluripotent
D) Embryonic stem cells are differentiated
E) Embryonic stem cells are apoptotic

A

Embryonic stem cells are pluripotent

How well did you know this?
1
Not at all
2
3
4
5
Perfectly
95
Q

Which of the following is NOT true of the assembly of actin filaments?
A) Polymerization is faster at the barbed end than at the pointed end.
B) Polymerization can occur at both the barbed and pointed ends of the filament.
C) ADP bound actin associates more readily than ATP-bound actin
D) Actin binding proteins can affect the polymerization rate
E) It begins with the formation of an aggregate of three actin monomers.

A

ADP bound actin associates more readily than ATP-bound actin

How well did you know this?
1
Not at all
2
3
4
5
Perfectly
96
Q

Which of the following statements about cloning by somatic cell nuclear transfer is false?
A) Somatic cell nuclear transfer was first carried out in frogs in the 1950s.
B) In the laboratory, the cloned egg is cultured to an early embryo stage and then transferred to a
foster mother.
C) The cloned embryo is genetically identical to the foster mother.
D) The nucleus of an adult somatic cell is transferred to an enucleated unfertilized egg.

A

The cloned embryo is genetically identical to the foster mother.

How well did you know this?
1
Not at all
2
3
4
5
Perfectly
97
Q
In which phase of the cell cycle in animal cells does one observe the accumulation of cyclin D which activates Cdk4 and Cdk6 and which in turn increases the synthesis of cyclin E which activates Cdk2 to phosphorylate proteins that cause the start of the next cell cycle phase?
A) G2
B) G1 
C) M 
D) D 
E) S
A

G1

How well did you know this?
1
Not at all
2
3
4
5
Perfectly
98
Q
Members of the Bcl-2 family of proteins are
A) all antiapoptotic.
B) either antiapoptotic or proapoptotic.
C) all proapoptotic.
D) all caspases.
A

either antiapoptotic or proapoptotic

How well did you know this?
1
Not at all
2
3
4
5
Perfectly
99
Q
Which of the following is formed by the assembly of dimers of proteins held together by a central coiled-coil domain?
A) Intermediate filaments
B) Microtubules
C) Actin filaments and microtubule
D) Actin filaments
E) Actin and Intermediate filaments
A

Intermediate filaments

How well did you know this?
1
Not at all
2
3
4
5
Perfectly
100
Q

Tumor suppressors are usually _______
A) factors that change in abundance during the cell cycle
B) proteins that form intermediate filaments
C) factors that inhibit or prevent cell proliferation
D) proteases that degrade cell components
E) members of a signaling pathway that stimulates growth when activated

A

factors that inhibit or prevent cell proliferation

How well did you know this?
1
Not at all
2
3
4
5
Perfectly
101
Q
Which of the following is activated by interaction with Apaf-1 with cytochrome c bound when the apoptotic pathway is activated?
A) Caspase-9
B) Bax
C) Ced-9
D) p53
E) Caspase-3
A

Caspase-9

102
Q

The normal function of the tumor suppressor protein Rb is to
A) inhibit progression through the G1 restriction point.
B) inhibit Cdk4/cyclin B activity.
C) inhibit Ras.
D) induce apoptosis.

A

inhibit progression through the G1 restriction point.

103
Q
Which of the following acts as a motor to move along microtubules towards the positive (+) end?
A) Mysosin
B) Vimentin
C) Dyneins
D) Kinesins
E) All of the above
A

Kinesins

104
Q

In the colon, one may observe a benign tumor or polyp referred to an adenoma. Why is this thought to occur?.
A) A cell has gotten a mutations in the gene encoding Rb
B) The cells have had no change to their genome but just starting growing
C) A cell has gotten a mutation in the gene encoding p53
D) Each cell in the tumor has a different mutation in an oncogene or tumor suppressor.
E) A cell has gotten a mutation in one or two genes that encodes one of the members of the Wnt pathway and/or the Ras/Raf pathway

A

A cell has gotten a mutation in one or two genes that encodes one of the members of the Wnt pathway and/or the Ras/Raf pathway

105
Q

Which of the following is NOT true about cancer?
A) Abnormal proliferation of cells
B) Usually develops from a single cell - clonal
C) Usually occurs due to one change or mutation in the cancerous cell
D) Can be caused by viruses
E) Can be caused by certain chemicals

A

Usually occurs due to one change or mutation in the cancerous cell

106
Q

Scientists have recently discovered how to reprogram adult skin cells back to a pluripotent state. How were the scientists able to do this?
A) By adding cyclins to the cells
B) By adding four hormones to the media
C) By mutating four genes in the chromosome of the skin cells
D) By deleting chromosome four of the skin cells
E) By introducing four genes into the skin cells that were observed to be expressed in embryonic stem cells

A

By introducing four genes into the skin cells that were observed to be expressed in embryonic stem cells

107
Q

The segregation of chromosomes to opposite ends of the cell during anaphase is mediated by which of the following?
A) The shortening of actin filaments attached to the chromosomes
B) Motor proteins acting on adjacent microtubules
C) The shorten of microtubules attached to the chromosomes
D) The disassembly of intermediate filaments
E) BandC

A

E- both:
Motor proteins acting on adjacent microtubules
The shorten of microtubules attached to the chromosomes

108
Q

When Cdks are active, they _____
A) Act as a transcription factor that cause the expression of genes needed for cell cycle progression
B) Form pores in the mitochondria which causes apoptosis
C) Act to add ubiquitin to proteins which leads to apoptosis
D) Act to as a protease that cleave proteins to they cause cell cycle progression
E) Act as a kinase that phosporylates proteins that in turn act to cause progression to the next cell cycle phase

A

Act as a kinase that phosporylates proteins that in turn act to cause progression to the next cell cycle phase

109
Q
Actin filaments play a major role in forming all of the following except \_\_\_\_\_
A) Microvilli
B) Pseudopods
C) Stress fibers
D) Focaladhesions
E) Cilia
A

Cilia

110
Q

To prevent cells with unreplicated DNA from passing through the G2 checkpoint, CHK1 and CHK2

\_\_\_\_\_\_\_ protein phosphatase Cdc25C which \_\_\_\_\_\_\_ its activity.
A) phosphorylate;inhibits
B) dephosphorylate; inhibits
C) phosphorylate; stimulates
D) dephosphorylate; stimulates
A

phosphorylate;inhibits

111
Q

]Which of the following types of cytoskeleton are often very dynamic i.e. change length rapidly and frequently?
A) Intermediate filaments
B) Actin filaments and microtubules
C) Intermediate filaments and microtubules
D) Microtubules
E) Actin filaments

A

Actin filaments and microtubules

112
Q
Which of the following plays a major role in causing apoptosis?
A) Cdks
B) Maturation promoting factor
C) Cyclins
D) Rb
E) Caspases
A

Caspases

113
Q

Which of the following is NOT a common cause of cancer?
A) Infection by a virus carrying a tumor suppressor
B) Exposure to radiation
C) Mutation that eliminates a tumor suppressor
D) Exposure to chemicals that damage DNA
E) Mutation of a proto-oncogene so that the gene produces an altered, but active protein

A

Infection by a virus carrying a tumor suppressor

114
Q

How does the release of Ca2+ ions activate muscle contraction?
A) Causes the release of additional A TP
B) Causes a protein complex binding to the actin filament to move thereby uncovering the myosin
head binding sites
C) Increases the affinity of the myosin head for actin
D) Causes the production of A TP
E) Causes the myosin head to cleave GTP at a slower

A

Causes a protein complex binding to the actin filament to move thereby uncovering the myosin

115
Q
Most cells in an adult animal are
A) undergoing meiosis.
B) stem cells.
C) in G0, or quiescent, phase.
D) actively proliferating.
A

in G0, or quiescent, phase.

116
Q

Which of the following statements is true about intermediate filaments?
A) Like microfilaments, they exhibit treadmilling.
B) Rather than consisting of a single type of protein, they can be made up of a number of different proteins.
C) They are involved in cell movement.
D) The basic structure of an intermediate filament protein is a globular head and a long -helical tail.

A

Rather than consisting of a single type of protein, they can be made up of a number of different proteins.

117
Q

Which of the following events is not part of apoptosis?
A) Fragmenting of the nucleus
B) Lysis of cell fragments and release of their contents
C) Fragmenting of chromosomal DNA
D) Fragmenting of the cell

A

Lysis of cell fragments and release of their contents

118
Q
Which of the following is a cellular protein that is activated in response to DNA damage and then stimulates the expression of a protein that can act to stop cell cycle progression and proteins that induce apoptosis?
A) Src
B) Ras
C) p53 
D) Cdk1 
E) Rb
A

p53

119
Q

A scientist constructed a mutant version of a kinase gene that encodes a protein that acts to in a pathway that signals the binding of a growth factor in rats. The mutated kinase protein produced from this gene lacks an inhibitory domain. What likely would happen if this gene was expressed in high levels in a normal rat skin cells?
A) Would cause the cells to grow slightly faster (proceed around cell cycle faster) and they would produce a few spots where cells are piled on top of each other
B) Would cause the growth rate of the cells to slow down
C) No effect - all cells would stop growing once they form a single layer of cells across the culture
dish
D) Would cause all the cells to become malignant
E) Would cause all the cells to undergo apoptosis

A

Would cause the cells to grow slightly faster (proceed around cell cycle faster) and they would produce a few spots where cells are piled on top of each other

120
Q
28. The activity of Cdk1 (cyclin dependent kinase) is regulated by \_\_\_\_\_
A) Changes its rate of degradation
B) Phosphorylation
C) Protein-protein interaction
D) BothAandB
E) BothBandC
A

Both B and C

phosphorylation and protein-protein interaction

121
Q
  1. The spindle assembly checkpoint prevents cell cycle progression until
    A) cyclin D accumulates to sufficient levels
    B) all the intermediate filaments assembly at each pole
    C) all the sperm have assembled their spindles
    D) any damaged DNA is repaired
    E) all the chromosomes are correctly aligned in the middle of the cell during metaphase
A

all the chromosomes are correctly aligned in the middle of the cell during metaphase

122
Q
30. Which of the following are chemicals or factors that are released from the mitochondria that act to stimulate apoptosis?
A) Cytochrome c
B) IAPs (inhibitors of apoptosis)
C) glucose
D) IAP inhibitors
E) A and D
A

E) A and D

Cytochrome C
IAP inhinitors

123
Q
  1. Which of the following is NOT true about maturation promoting factor (MPF)?
    A) Composed of two proteins (cyclin B and a cyclin dependent kinase 1)
    B) Promotes progression from G2 to M phase of frog oocyctes
    C) The levels of the kinase subunit go up and down with the cell cycle phase
    D) Activity is cyclic with highest activity at the start of M phase
    E) When active, maturation promoting factor acts to phosphorylate other protein
A

The levels of the kinase subunit go up and down with the cell cycle phase

124
Q

List three properties that distinguish cancerous cells from normal cells.

A

Lack of density-dependent inhibition of proliferation
Lack of differentiation
Reduced requirement for extracellular growth factors
Abnormal surface interactions
Failure to undergo apoptosis
Promote angiogenesis

125
Q

Describe three ways a proto-oncogene can be converted into an oncogene or give three examples of the conversion of a proto-oncogene to an oncogene.

A

Mutation that causes the protein to be constitutive on
Mutation that eliminates a domain that inhibits the activity of the proto-oncogene product Mutation that causes overexpression of the proto-oncogene
Chromosomal rearrangement/ translocation

126
Q

Humans are able to replace most of the differentiated cells that make up its various tissues, but the differentiated cells in different tissues are replaced by different mechanisms. Explain two mechanisms or means by which new differentiated cells will be generated. Provide an example of a tissue or cell type that uses each mechanism.

A

a. Differentiated cells of tissue are still able to proliferate. See in fibroblasts (skin), endothelial cells of blood vessels, smooth muscle cells, liver and pancreas.
b. New differentiated cells are continuously produced from tissue specific stem cells. Examples include red and white blood cells, epithelial cells of intestine, skin
c. New differentiated cells are produced from a few dormant adult stem cells. Examples include skeletal muscles, most other organs/tissues (heart, lung, brain, etc).

127
Q

Which of the following is a modification of chromatin known to affect gene expression?
A) Phosphorylation of adenine bases of AT pairs of nucleotides
B) Acetylation of N-terminal tails of histone H3 and H4
C) Methylation of C-terminal tails of histone H1
D) Acetylation of the first thymine base of thymine nucleotide pairs in DNA
E) Addition of sugars to general transcription factor TFIIB

A

Acetylation of N-terminal tails of histone H3 and H4

128
Q

The splicing out of introns from mRNAs in mammals is catalyzed and accomplished by _____
A) RNA polymerase
B) by a single separate RNA
C) a single splicing protein
D) by a complex of several RNAs and proteins
E) the mRNA itself

A

a complex of several RNAs and proteins

129
Q
Which of the following amino acids is not commonly phosphorylated to regulate protein activity?
 A) Tryptophan
B) Threonine
C) Tyrosine
D) Serine
A

Tryptophan

130
Q

Which of the following describes the mechanism of translational initiation in prokaryotes?
A) The large 50S subunit + 3 initiation factors binds to a Shine-Dalgarno sequence downstream (3’)
of the AUG start codon
B) The small 30S subunit + 3 initiation factors binds to a Shine-Dalgarno sequence upstream (5’) of the AUG start codon
C) The large subunit + 12 initiation factors scans from the 3’ cap to the first AUG
D) The small subunit + 12 initiation factors scans from the 5’ cap to the first AUG
E) The 3 initiation factors bind to the Shine-Dalgarno sequence far from the AUG start codon and then interact with the ribosome through a mediator complex

A

The small 30S subunit + 3 initiation factors binds to a Shine-Dalgarno sequence upstream (5’) of the AUG start codon

131
Q
Where do endocytic vesicles that bring in material from the outside of the cell such as LDL particlesgo after leaving the plasma membrane?
A) To the endosome
B) To the endoplasmic reticulum (ER)
C) To the Golgi
D) To the mitochondria
E) To the ERGIC
A

To the endosome

132
Q

Put the steps in processing of an eukaryotic mRNA in the order in which they occur inside the cell.
A) Poly A tail addition, 5’ cap addition, splicing out of introns
B) Splicing out of introns, 5’ cap addition, poly A tail addition
C) No processing will occur because translation begins immediately after its synthesis is begun
D) Splicing out of introns, poly A tail addition, 5’ cap addition
E) 5’ cap addition, poly A tail addition, splicing out of introns

A

5’ cap addition, poly A tail addition, splicing out of introns

133
Q

A difference between transcription in eukaryotes verse prokaryotes is that
A) eukaryotes do not require a large group of general transcription factors to bind to the promoter
B) regulatory elements bound by specific transcriptional factors can be located a much longer distance away from the promoter
C) specific transcription factors act mostly as repressors in eukaryotes
D) eukaryotic RNA polymerases are only composed of 3 subunits while prokaryotic RNA
polymerases are composed of 9 subunits
E) transcription and translation occurs at the same time in eukaryotes

A

regulatory elements bound by specific transcriptional factors can be located a much longer distance away from the promotor

134
Q
In translation, mRNAs are read in the \_\_\_\_\_\_\_ direction, and polypeptide chains are synthesized from the \_\_\_\_\_\_\_ends.
A) 5 to 3; amino to the carboxyl
B) 3 to 5; amino to the carboxyl
C) 3 to 5; carboxyl to the amino
D) 5 to 3; carboxyl to the amino
A

5 to 3; amino to the carboxyl

135
Q

The addition of either the lipid myristic acid, prenyl group or palmitic acid occurs in the _____ and targets the protein to ____.
A) occurs in the ER and targets the protein to the lysosome
B) occurs in the ER and targets the protein to the mitochondria
C) occurs in the cytosol and targets the protein to the cytosolic face of the plasma membrane
D) occurs in the Golgi and targets the protein to the endosome
E) occurs in the cytosol and targets the protein to the lysosome

A

occurs in the cytosol and targets the protein to the cytosolic face of the plasma membrane

136
Q

What can the addition of ubiquitin to a protein do?
A) Target the protein for degradation by the proteosome
B) Target the protein to the ER
C) Target the protein to the lysosome
D) Target the protein to the plasma membrane
E) Target the protein to the Golgi

A

Target the protein for degradation by the proteosome

137
Q

The genetic code indicates that six different codons code for the amino acid leucine.
following statements is true about the usage of the six different codons for leucine by organisms?
A) The six different codons are used in different frequencies but that frequency is the same in all
organisms.
B) A given organism would only use one of the six codons, but there is variation in which one is used
C) There is variation in the frequency of the use of the six codons between organisms.
D) All organisms only use one of the six possible codons.
E) All organisms use all six of the possible codons equally.

A

There is variation in the frequency of the use of the six codons between organisms.

138
Q
Most cellular lipids are synthesized in the \_\_\_\_\_\_.
A) Lysosome
B) Golgi
C) Rough ER membrane
D) Cytosol
E) Smooth ER membrane
A

Smooth ER membrane

139
Q

When is the signal sequence usually cleaved off a protein destined to be secreted?
A) While it is in the ER before it is pumped into the ER
B) Shortly after the ribosome starts to pump the protein through a translocon into the ER
C) In the Golgi
D) In the secretory vesicle just before secretion
E) After the entire protein is pumped into the ER

A

Shortly after the ribosome starts to pump the protein through a translocon into the ER

140
Q

Which of the following describes the origin of the proteins in the mitochondria?
A) All are synthesized in the mitochondria
B) All are synthesized in the cytosol and then imported into the mitochondria
C) All are transported to the mitochondria via vesicles from the Golgi
D) A few are made in the mitochondria but most are synthesized in cytosol and then imported into the mitochondria
E) A few are synthesized in the cytosol and then then imported into the mitochondria but most are transported via vesicles from the Golgi to the mitochondria

A

A few are made in the mitochondria but most are synthesized in cytosol and then imported into the mitochondria

141
Q
Which of the following is an example of a DNA binding domain found in specific transcription factors?
A) Calcium knuckle
B) Zinc finger
C) Helix-turn-helix
D) -sheet spiral
E) BandC
A

zinc finger and helix-turn-helix

142
Q

If the analysis of amino acids in a protein indicated that the protein had cleavable signal sequence and three transmembrane domains then one could also predict which of the following about the orientation of the protein in the ER membrane?
A) One would predict that the C-terminus would be located in the lumen of the ER but would not
know the orientation of the N-terminus.
B) One would predict that the N-terminus would be located in the lumen of the ER and the C-terminus would be located in the cytosol.
C) One would predict that the N-terminus would be located in the cytosol, but would not know the orientation of the C terminus.
D) One would predict that the N-terminus would be located in the cytosol of the ER and the C-terminus would be located in the lumen of the ER.
E) One would predict that both the N-terminus and C-terminus would be located in the lumen of the ER

A

One would predict that the N-terminus would be located in the lumen of the ER and the
C-terminus would be located in the cytosol.

143
Q

Which of the following is NOT observed in eukaryotic cells?
A) Four to five different RNA polymerases
B) Mediator complex mediate the interaction of specific transcription factors with RNA polymerase
C) Enhancers being up to 100 kB upstream or downstream of the promoter
D) Binding of the RNA polymerase to the promoter being assisted by a large complex of general
transcription factors
E) Polycistronic mRNA with multiple genes on it that code for different proteins

A

Polycistronic mRNA with multiple genes on it that code for different proteins

144
Q

Clathrin-coated vesicles are involved in
A) the process of secretion.
B) the uptake of extracellular molecules by endocytosis and the transport of molecules from the trans
Golgi network to the lysosomes and the plasma membrane.
C) the retrieval of ER resident proteins from the cis-Golgi or the ER-Golgi intermediate
compartment.
D) transport from the ER to the Golgi.

A

the uptake of extracellular molecules by endocytosis and the transport of molecules from the trans
Golgi network to the lysosomes and the plasma membrane.

145
Q
Proteins that facilitate the folding of other proteins are called
A) binding proteins.
B) escorts.
C) chaperones.
D) foldases.
A

chaperones

146
Q

Which of the following describes how N-linked oligosaccharides are added to proteins and modified?
A) The sugars are added one at a time in the Golgi
B) The sugars are added one at a time in the ER
C) A common 14 sugar oligosaccharide is added in the ER and then it is modified in the ER and the Golgi
D) A common 4 sugar oligosaccharide is added in the ER and then it is modified in the Golgi
E) A common 9 sugar oligosaccharide is added in the Golgi and then it is modified in the Golgi

A

A common 14 sugar oligosaccharide is added in the ER and then it is modified in the ER and the Golgi

147
Q

Which of the following is NOT true about the role of sigma factors in transcription?
A) It is responsible for RNA polymerase recognizing and binding to promoter sequences.
B) It interacts directly with RNA polymerase.
C) Each prokaryotic cell has only one type of sigma factor.
D) Some are used to mediate the coordinate regulation of a set of genes.
E) It is required for initiation of transcription of genes in prokaryotes, but not in eukaryotes.

A

Each prokaryotic cell has only one type of sigma factor.

148
Q

A large complex of general transcription factors including TFIIB being necessary for the RNA polymerase to bind to the promoter is observed _____
A) only in prokaryotic cells.
B) only in eukaryotic cells.
C) in both prokaryotic and eukaryotic cells.
D) in neither prokaryotic and eukaryotic cells.

A

only in eukaryotic cells

149
Q
In which of the following is the import of proteins into the organelle usually mediated by a sequence on the amino terminus of the protein?
A) Import into the nucleus
B) Import into the mitochondria
C) Import into the ER
D) Import into the chloroplast
E) B,C,andD
A

E-
Import into the mitochondria
Import into the ER
Import into the chloroplast

150
Q

Enzymes involved in processing of RNA associate with ________ after it is phosphorylated.
A) the CTD (carboxy terminal domain) of RNA polymerase II
B) the Mediator complex
C) sigma factor 70
D) the TFIIG complex
E) the TATA binding protein

A

the CTD (carboxy terminal domain) of RNA polymerase II

151
Q
Which of the following is a type of protein that plays a role in protein synthesis by ribosomes, coated vesicle formation and vesicle fusion with its target membrane?
A) Riboprotein
B) Glycosylated protein
C) Helix-turn-Helix domain protein
D) G protein
E) Coiled-coil domain protein
A

G Protein

152
Q
Rabs and SNAREs are involved in
A) Transcriptional initiation in eukaryotes
B) Electron transport
C) Chromatin formation
D) Vesicular fusion with target membrane
E) Translation
A

Vesicular fusion with target membrane

153
Q

Which of the following describes how the new amino acid is linked to the growing peptide chain during the elongation cycle of translation?
A) The amino acid attached to the tRNA in the A site is linked to the peptide chain attached to the
tRNA in the E site
B) The peptide attached to the tRNA in the E site is linked to the amino acid attached to the tRNA in
the P site
C) The peptide attached to the ribosome is linked to the amino acid attached to the tRNA in the P site
D) The amino acid attached to the tRNA in the E site is linked to the peptide chain attached to the
tRNA in the P site
E) The peptide attached to the tRNA in the P site is linked to the amino acid attached to the tRNA in the A site

A

The peptide attached to the tRNA in the P site is linked to the amino acid attached to the tRNA in the A site

154
Q

A scientist is studying a mutant strain of E. coli bacteria that is now making some beta-galactosidase
activity (LacZ) even when lactose is not present. This strain could have in a mutation in which of the following genes or regulator sequences?
A) LacI gene
B) CAP gene
C) LacI operator site
D) CAP site near lac promoter
E) AorC

A

E-

Lacl gene or Lacl operator site

155
Q
Which of the following is not a possible destination for protein-containing vesicles leaving the Golgi apparatus?
A) The plasma membrane
B) The exterior of the cell
C) Lysosomes
D) Mitochondria
A

Mitochondria

156
Q

Explain the route that a secreted protein would travel from its synthesis to its secretion from the cell?

A

The protein would start to be synthesized by a ribosomes and then if the protein would have a N-terminal signal sequence it would go the ribosome to go to the ER membrane and then the synthesis would begin again and the polypeptide would be pumped into the ER lumen. The signal sequence would be cleaved off. After the protein is completely synthesized, the protein may be modified and then would be transported in a vesicle to the ER-Golgi intermediate compartment and then to the cis face of Golgi stacks. The protein would pass through the Golgi stacks. In the trans Golgi network the protein would be bound by a receptor sitting where a vesicle is forming and that is destined to go to the plasma membrane. Then that vesicle would fuse with the plasma membrane and release the protein to the outside of the cell.

157
Q

Describe how a protein in the Golgi is selectively transported to a specific organelle like the lysosome. Mention what the protein to be transported is recognized.

A

The protein to be transported would have a specific short amino acid sequence in it or a modification of an olgiosacchariede (such as phosphorylation of mannose sugars for lysosomal proteins) that would be by bound by receptor protein that is present in the the area of the trans face of the golgi where a vesicle is forming to go to that organelle. The receptor protein is a transmembrane protein and interacts with a complex that helps to assembly a coat and other proteins that direct the transport of the protein to the specific organelle. The vesicle then buds off carrying the bound proteins and the vesicle has specific protein that allow it to fuse with the specific target membrane.

158
Q

a. How are amino acids attached to tRNAs?

b. There are many different tRNAs in a cell, how does the correct amino acid get added to the correct tRNA?

A

a. Added by enzyme called aminoacyl tRNA synthetase. It adds the amino acid to the 3’OH of the 3’ adenine on tRNA.
b. EAch tRNA has one or two specific enzymes (aminoacyl-tRNA synthases) in the cell that recognize certain tRNA by recognizing a specific sequences in the tRNA and then adds the correct or corresponding amino acid.

159
Q

Explain how specific transcription factor binding to enhancers can affect transcription initiation in eukaryotes. Mention the other factors with which they interact and their effect on DNA.

A

Specific transcription factors bind to specific enhancer sequences that may be far from the promoter. They cause the DNA to loop out so that they can interact with the general transcription factors that assemble on the TATA. Specific transcription factors interact with the mediator complex to interact which interacts with the general transcription factors and RNA pol II. Specific transcription factors also help to recruit chromatin remodeling proteins to activate that portion of DNA by acetylating histone tails.

160
Q

Which of the following is NOT true about mutations?
A) Mutations can be caused by mistakes by DNA polymerase during replication
B) Some types of mutations alter base pairing
C) Some types of mutations can cause polymerases acting on that strand to stall
D) Regions of the genome that are actively transcribed are more prone to mutation
E) Mutations can be caused by chemical alteration of bases

A

Regions of the genome that are actively transcribed are more prone to mutation

161
Q

What is different about the export of mRNA from the transport of other proteins and RNAs in and out of the nucleus?
A) Transported through the perinuclear space
B) No importin or exportin factors play a role in the transport
C) CTP is cleaved instead of GTP
D) Does not go through the nuclear pore
E) Ran G protein is not involved

A

Ran G protein is not involved

162
Q

Which of the following is NOT true about centromeres
A) A part is single stranded
B) Site of attachment of microtubules for segregation of sister chromatids
C) Point of attachment of sister chromatids
D) A region of heterochromatin
E) Composed of repetitive DNA elements

A

A part is single stranded

163
Q

Which of the following is NOT true about replication?
A) Short stretches of RNA are synthesized on the lagging strand
B) The sliding clamp protein increases the association of the DNA polymerase with the DNA strand so
that the DNA polymerase will continue to synthesis a longer stretch of DNA before falling off
C) There is a DNA polymerases on each strand and both are tethered to the same protein (clamp
loading protein)
D) New nucleotides are added to the 5’ end of the new strand being synthesized so that synthesis can
occur in a 3’ to 5’ direction
E) More than one DNA polymerase is involved

A

New nucleotides are added to the 5’ end of the new strand being synthesized so that synthesis can
occur in a 3’ to 5’ direction

164
Q
Fill in the blanks in order to use the terms correctly to describe the relationship between two genes. The genes Xyz and Abc are \_\_\_\_\_\_\_ because 90% \_\_\_\_\_\_\_ was observed in a sequence alignment between the DNA sequences for the two genes.
A) likely homologous, homology
B) likely homologous, identity
C) identical, similarity
D) identical, homology
E) similar, homology
A

likely homologous, identity

165
Q

The major pathway for molecules to pass into and out of the nucleus is through
A) porin channels in the nuclear envelope membranes.
B) nuclear pore complexes.
C) gap junctions between the nuclear envelope membranes.
D) diffusion through the membrane bilayers of the nuclear envelope.

A

nuclear pore complexes.

166
Q

The nucleolus is
A) Site in nucleus where introns are spliced out of mRNA
B) Site of synthesis and processing of rRNA and assembly of ribosomes
C) Site of attachment of microtubules for segregation during mitosis or meiosis
D) Site of transport of proteins into and out of nucleus
E) Site in nucleus where telomeres are located

A

Site of synthesis and processing of rRNA and assembly of ribosomes

167
Q

You are analyzing the newly sequenced genome of the organism Drosphilia nebraski and discover the
presence of multiple similar DNA sequences that appear to be genes that encode similar proteins. sequences would be called _______.
A) SINEs
B) Pseudogenes
C) A gene family
D) Single sequence repeats
E) Introns

A

A gene family

168
Q

Unweighted Pair Group method with Arithmetic mean (UPGMA) is used to __________
A) to determine if two sequences are identical
B) to determine if two sequences are similar
C) to determine how to build a phylogenetic tree between related sequences
D) to predict if a sequence is a gene
E) All of the above

A

to determine how to build a phylogenetic tree between related sequences

169
Q
Homologous recombination plays a role in repairing which of the following types of alterations of or damages to DNA?
A) Double strand break
B) Depurination
C) Deamination
D) Alkylation
E) Thymine dimers
A

double strand break

170
Q
You have a DNA sequence for gene Z and want to see if similar sequences exist in other organisms. Which of the following is the program at NCBI that one can use to search for similar sequences in the NCBI genetic information databases?
A) BLAST
B) Googlegene
C) UPGMA
D) Pubmed
E) Gene
A

BLAST

171
Q
The outer nuclear membrane is contiguous with
A) the endoplasmic reticulum.
B) the plasma membrane.
C) the nuclear lamina.
D) the Golgi apparatus.
A

The ER

172
Q

Which of the following is NOT true about telomeres?
A) Very end is single-stranded and forms a loop
B) Prevents degradation of DNA
C) Composed of a simple sequence repeat element
D) Found of both ends of chromosomes in eukaryotes
E) Number of repeats usually increases as a cells divides and ages

A

Number of repeats usually increases as a cells divides and ages

173
Q
Radioactive materials used to produce X-rays, and nuclear energy produce ionizing radiation such as gamma rays which is mutagenic. Which of the following is a mutagenic alteration of DNA that ionizing radiation promotes?
A) The formation of pyrimidine dimers
B) Deamination of bases
C) Depuration of nucleotides
D) Double strand breaks of DNA
E) All of the above
A

Double strand breaks of DNA

174
Q

If a cell has a mismatch in its DNA after replication, it can repair it using the mismatch repair system. In this system, the repair enzymes selectively cleave nucleotides from the newly synthesized strand and then use the old strand as a template to replace the removed nucleotides. How are the mismatch repair enzymes able to distinguish the old strand from the newly synthesized strand?
A) The old strand will have methylated C or methylated A bases
B) The new strand will be have apurnic nucleotides
C) The old strand will deaminated G nucleotides
D) The old strand will contain uracil bases while the new strand will not
E) The new strand will have aminated T

A

The old strand will have methylated C or methylated A bases

175
Q

The transport of proteins and RNAs into and out of the nucleus is powered by ______?
A) the phosphorylation of importins or exportins
B) a proton gradient across the nuclear membrane
C) the cleavage of CTP by the nuclear pore
D) the cleavage of GTP to GDP by a G protein
E) All of the above

A

the cleavage of GTP to GDP by a G protein

176
Q

Written below is a citation for journal article. Which of the following is true
Finck-Barbancon, V., T. L. Yahr, and D. W. Frank. 1998. Identification and characterization of SpcU, a chaperone required for efficient secretion of the ExoU cytotoxin. J Bacteriol 180: 6224-31.
A) The volume is J Bacteriol
B) The issue is 1998
C) This issue is 6224
D) The volume is 6224
E) The volume is 180

A

the volume is 180

177
Q

Which of the following is the relationship between two properties of genomes that shows the highest correlation?
A) The inverse or negative relationship between log of genome size and the number of genes
B) The positive relationship between percent protein coding and the number of chromosomes
C) The positive relationship between log of genome size and the number of chromosomes
D) The inverse or negative relationship between log of genome size and percent protein coding
E) The inverse or negative relationship between number of chromosomes and number of genes

A

The inverse or negative relationship between log of genome size and percent protein coding

178
Q

How is an eukaryotic chromosome replicated?
A) Starts at one end and goes in one direction from the start site
B) Starts at multiple sites per chromosome and proceeds in one direction from each
C) Starts at only one site per chromosome and proceeds in both directions from each start site
D) Starts at multiple sites per chromosome and proceeds in both directions from each start site
E) Starts at only one site per chromosome and proceeds in one directions from each start site until goes
all the way around the circular chromosome

A

Starts at multiple sites per chromosome and proceeds in both directions from each start site

179
Q

The principal element providing structural support for the shape of the nucleus is _________ of the nuclear envelope.
A) the chromatin scaffolding proteins for each chromosome
B) a meshwork of branched polysaccharide chains located in the perinuclear space
C) a meshwork of intermediate filaments attached to the interior face of the nuclear envelope
D) a meshwork of straight polysaccharide chains attached to the interior of the nuclear envelope
E) a meshwork of microtubules attached to the cytoplasmic face of nuclear envelope

A

a meshwork of intermediate filaments attached to the interior face of the nuclear envelope

180
Q
Fill in the blanks. One major type of molecule that is transported out of the nucleus is \_\_\_\_\_\_\_\_ and one major type of molecule that is transported into the nucleus is \_\_\_\_\_\_\_\_\_
A) DNA, processed mRNA
B) processed mRNA, proteins
C) unprocessed mRNA, processed mRNA
D) proteins, unprocessed mRNA
E) proteins, DNA
A

processed mRNA, proteins

181
Q
What expectation value or E value would suggest that the query and a hit are homologous if one was doing a protein blast?
A) 1.0e5
B) 1
C) 1.0e-50 
D) 100
A

1.0e-50

182
Q

Which of the following is the type of DNA element that makes up the largest percentage of the DNA in the human genome?
A) Retrotransposons (SINEs, LINEs, etc) and retroviral-like elements
B) Introns
C) Spacers between genes
D) Protein coding sequences
E) Simple-sequence repeats

A

Retrotransposons (SINEs, LINEs, etc) and retroviral-like elements

183
Q

Humans with the disease xeroderma pigmentosum (XP) have a defect in nucleotide-excision repair which causes them to be especially sensitive to ________ because nucleotide-excision repair is the chief mean that humans use to repair the alterations it causes.
A) errors in replication caused by DNA polymerase
B) depurination
C) ultraviolet light
D) deaminating agents
E) iozinzing radiation

A

UV light

184
Q

Which of the following is true about how genomes change?
A) Ionizing radiation cause depurination
B) Ultraviolet (UV) light causes double strand breaks
C) All methylations of bases are mutagenic.
D) Lose of amine groups (deamination) from bases is mutagenic
E) Pyrimidine dimers form between nonadjacent adenine nucleotides

A

Lose of amine groups (deamination) from bases is mutagenic

185
Q
  1. If one was trying to identify genes in a newly sequenced bacterial genome, one could look for ______
    A) for every ATGs, a new gene would begin at each ATG site
    B) the regions that show 100% identity with another bacteria
    C) transcriptional initiation sites
    D) long open reading frames (over 90 bp) lacking a stop codon
    E) the gene here sign
A

long open reading frames (over 90 bp) lacking a stop codon

186
Q

Describe how the DNA in an eukaryotic chromosomes is packaged to form the chromatin seen in cells.

A

The DNA is wrapped around histone octomers to form nucleosomes, the nucleosomes can link in a spiral arrangements to form 30 nm fibers. The 30 nm fibers are attached in loops to a chromosome scallfold. The DNA can be wrapped more tightly so that it is no longer it can not be transcribed or replication (heterochromatin).

187
Q

A couple of sequences for the human genome have been published and those produced some unexpected results. What are two questions that scientists still have about human genome?

A

Why so much repetitive DNA elements including transposons? Do they provide any benefit?
What is the function of all the proteins with unknown function?
How do the small differences between the chimpanzee and human genome cause the difference between species?
Why so few genes?
Role and value of introns?

188
Q

Describe the process by which proteins enter the nucleus and how the factors involved are recycled for later reuse?

A

Importin binds to the nuclear localization signal sequence on protein. Importin and bound protein gets transported into nucleus through nuclear pore. Ran-GTP binds to importin causing it to release the transported protein into the nucleus. Ran-GTP-importin complex are transported out of the nucleus and then Ran GAP stimulates Ran to cleave GTP which cause Ran to release the importin.

189
Q

Explain one of the methodologies used to determine the entire sequence the human genome. You need to describe how the genome was broken up for sequencing and how the results of individual sequencing reactions was assembled to get the final complete sequence. You do not need to describe how DNA is sequenced.

A

A. Shotgun cloning: The whole genome was broken up into small 1 kB peices and randonly cloned into plasmid vectors. Hundreds or thousands of differents plasmid with inserts were sequenced. Use computer to link and assembly Different DNA fragments together into one DNA molecule.
B. BAC method. First create BAC library will large DNA fragments (~100 kb). Order and select BACs (bacterial artificial chromosomes) to use. Sequence DNA in selected BACs by shotgun cloning.

190
Q

The expression of a gene can be silenced in eukaryotic cells by _____
A) by laser microscopy
B) by cloning the gene into an expression plasmid vector
C) in situ hybridization
D) RT PCR
E) introducing a short interfering RNA into the cells

A

introducing a short interfering RNA into the cells

191
Q

One method to make transgenic mice would be to
A) add isolated DNA as a calcium phosphate precipitate to all the cells of a mouse.
B) inject whole nuclei into embryonic stem cells
C) infect mice with viruses carrying the gene.
D) inject DNA containing the gene into the pronucleus of a fertilized egg.

A

inject DNA containing the gene into the pronucleus of a fertilized egg.

192
Q

RNA is hypothesized to have been the genetic material of the original cell because
A) RNA contains uracil which is more stable than thymine
B) RNA contains dideoxyribose sugars which are more stable
C) RNA can catalyze chemical reactions like polymerization
D) RNA is double stranded while DNA is not.
E) All of the above

A

RNA can catalyze chemical reactions like polymerization

193
Q

You are analyzing the types of fatty acids in the cell membrane of two shrimps. One shrimp was isolated from the water near the arctic circle was grown in a temperature of 10°C and the other shrimp was isolated from the Caribbean ocean and was grown at a temperature of 30°C. Which of the following is a difference you would expect to see in the type of fatty acids found in each shrimp?
A) The arctic shrimp will have more proteins in their membrane
B) The arctic shrimp will have a higher ratio of unsaturated fatty acids
C) The arctic shrimp will have more sphinogomyelin
D) The arctic shrimp will have a higher ratio of saturated fatty acids
E) The arctic shrimp will have less cholesterol

A

The arctic shrimp will have a higher ratio of unsaturated fatty acids

194
Q

You are conducting a microarray experiment and have labeled the cDNAs generated from the A cell line with a red dye and the cDNA generated from the B cell line with the green dye. For gene spot 12, you observe that the color was green, what does that mean?
A) The scientists did not generate any cDNAs from the A cell line
B) Gene 12 is only found in the B cells
C) More copies of the mRNAs for gene 12 were present in the B cells than in the A cells
D) The B cells contained more copies of the gene 12 in their chromosome than the A cells
E) The protein from gene 12 is only found in B cells

A

More copies of the mRNAs for gene 12 were present in the B cells than in the A cells

195
Q

Steroid hormones principally enter cells via ______.
A) Active transport through a transport protein that cleaves ATP
B) Diffusion through the phospholipid bilayer
C) Active transport through a symport transport protein that also transport H+ ions in
D) Through a ion channel protein
E) C and D

A

Diffusion through the phospholipid bilayer

196
Q

Which of the following is NOT a difference between RNA and DNA?
A) The bases found in each
B) The sugar found in the nucleotides of each
C) The linkage between the nucleotides in each
D) The ability to act like an enzyme
E) A and B

A

The linkage between the nucleotides in each

197
Q

Transmission electron microscopy is used to _______
A) observe the fine details of intracellular cell structures such as organelles and membranes
B) to detect the levels of a specific protein with an antibody
C) to knock out the expression of a gene
D) to identify and separate a specific class of cells from a mixed population
E) observe where a fluorescent tagged protein is located

A

observe the fine details of intracellular cell structures such as organelles and membranes

198
Q
Which of the following is a method to test whether two different cellular proteins interact?
A)	Microarray
B)	Yeast two-hybrid system
C)	Real time PCR
D)	Southern blot
E)	2D gel electrophoresis
A

Yeast two-hybrid system

199
Q

Active transport differs from facilitated diffusion in that
A) active transport involves the transport of molecules up their concentration gradient.
B) ions are not transported via active transport.
C) active transport involves a conformational change in the transport molecule.
D) active transport requires a protein component, whereas facilitated diffusion occurs by simple diffusion through the plasma membrane.

A

active transport involves the transport of molecules up their concentration gradient.

200
Q

Expression of a human protein in a bacterial cell is best done with human
A) genomic DNA in a yeast artificial chromosome.
B) cDNA in a bacterial expression vector.
C) genomic DNA in a bacterial expression vector.
D) cDNA in a yeast artificial chromosome.

A

cDNA in a bacterial expression vector.

201
Q

Which of the following is the best description of the movement observed of integral membrane proteins?
A) Some integral membrane proteins can diffuse laterally through the membrane while others are held in place and do not move laterally and proteins do not flip their orientation in the membrane.
B) All integral membrane proteins will diffuse lateral through the membrane.
C) All integral membrane proteins can flip their orientation in the membrane (portion on outside verse inside).
D) Integral membrane proteins do not move in any manner in the membrane.
E) B and C

A

Some integral membrane proteins can diffuse laterally through the membrane while others are held in place and do not move laterally and proteins do not flip their orientation in the membrane.

202
Q
What is used to generate double strand DNA (a cDNA) from a mRNA in a microarray or RT PCR procedure?
A)	siRNA
B)	DNA ligase
C)	Taq DNA polymerase
D)	Reverse transcriptase
E)	A restriction endonuclease
A

Reverse transcriptase

203
Q
The Western blot technique detects a specific \_\_\_\_ .
A)	protein with an antibody
B)	protein with a DNA probe
C)	DNA with an DNA probe
D)	RNA with a DNA probe
E)	DNA with a RNA probe
A

protein with an antibody

204
Q
Which of the following can act to both increase and decrease the fluidity of membranes depending on the temperature and its concentration?
A)	Sphingolipid
B)	Membrane proteins
C)	Cholesterol
D)	Unsaturated fatty acids
E)	Saturated fatty acids
A

Cholesterol

205
Q

If the concentration of Na+ and K+ ions was 10 mM and 140 mM on the inside of an animal cell and 145 mM and 5 mM, respectively, on the outside of the cell, then which of the following describes a way that potassium ions would enter the cell under these conditions?
A) Potassium ions (K+) could enter the cell by facilitated diffusion through a pore or channel protein
B) Potassium ion (K+) could enter the cell by a transporter that cleaved ATP to drive the transport
C) Potassium ion (K+) could enter the cell by a symport that transported a K+ ion in as a Na+ ion was also transported inside the cell
D) B and C
E) A, B and C

A
D- B and C
Potassium ion (K+) could enter the cell by a transporter that cleaved ATP to drive the transport
Potassium ion (K+) could enter the cell by a symport that transported a K+ ion in as a Na+ ion was also transported inside the cell
206
Q
Which of the following is not correctly matched?
A)	Drosophila - frog
B)	Saccharamyces cerevisiae  - yeast
C)	Arabidopsis  - plant
D)	C. elegans - nematode
E)	E. coli  - bacteria
A

Drosophila - frog

207
Q
Which of the following is a method that could be used to isolate just CD4+ T cells from all the other type of cells in a sample of blood?
A)	Density-gradient centrifugation
B)	Transmission electron microscopy
C)	2D gel electrophoresis
D)	Western blot
E)	Flow cytometry and FACS analysis
A

Flow cytometry and FACS analysis

208
Q

Which of the following is a difference between glycogen and cellulose?
A) Type of sugar
B) Sugar is modified in glucose
C) α verse β bond linking sugars together
D) Cellulose is branched, while glycogen is not
E) A and B

A

α verse β bond linking sugars together

209
Q
Hydrogen bonding is frequently observed in which of the following?
A)	Nucleic acids
B)	Proteins
C)	Carbohydrates
D)	Triacylglycerols or fats
E)	A, B and C
A

E
Nucleic Acids
Proteins
Carbohydrates

210
Q
Which of the following is thought to have arisen by endosymbiosis?
A)	Golgi
B)	Endoplasmic reticulum
C)	Chloroplast
D)	Mitochondria
E)	C and D
A

E
Chloroplast
Mitochondria

211
Q

Which of the following is true about triacylglycerols or fats?
A) Linkage between fatty acids can be alpha or beta
B) Contain two fatty acids linked to a glycerol molecule
C) Are amphipathic
D) Located in aggregates or droplets in the cytoplasm of cells
E) All of the above

A

Located in aggregates or droplets in the cytoplasm of cells

212
Q

You have done a hydrophobicity plot of the amino acids of a protein and observe an region of 21 amino acids with a very high average hydrophobicity. You would conclude that ______
A) this region is likely on the surface or exterior of the folded protein.
B) this region likely forms a double helix between two complementary strands
C) this region likely forms a beta sheet secondary structure.
D) this region likely binds to a sugar
E) this region is likely a transmembrane domain

A

this region is likely a transmembrane domain

213
Q

Which of the following is NOT an advantage of Next generation pyrosequencing as compared to Sanger dideoxy sequencing?
A) Next generation does not require fragments of genomic DNA be cloned into a plasmid for sequencing
B) Next generation sequencing allows one to perform millions of sequencing reactions at one time in one plate
C) Next generation sequencing produces longer reads from each reaction (over 1000 base pairs)
D) Next generation sequencing is able to measure the addition of a nucleotide in real time (measures PPi released due to the addition)
E) Next generation sequencing requires less template DNA

A

Next generation sequencing produces longer reads from each reaction (over 1000 base pairs)

214
Q
Base pairing between an adenine base in one strand to an thymine base in the complementary strand is mediated by \_\_\_\_\_\_\_\_\_
A)	three hydrogen bonds
B)	three ionic bonds
C)	two ionic bonds
D)	two hydrophobic interactions
E)	two hydrogen bonds
A

two hydrogen bonds

215
Q

Which of the following is true about the phospholipids one would observe in the membranes of an eukaryotic organism?
A) The type of phospholipids found in the membranes would vary by the cell type but would be the same within all the organelles of the cell.
B) The type of phospholipids found would vary in the different cell types and in the organelles of the different cell types but would be the same in both halves of each membrane.
C) All the membranes in the organism including the membranes of organelles of cells would contain the same ratio of different phospholipids.
D) The type of phosphoplipids found would vary in the membranes of the different cell types, in the membranes of the different organelles and in the two halves of each membrane.

E) All the membranes in the organism including the membranes of organelles would contain only one type of phospholipid.

A

The type of phosphoplipids found would vary in the membranes of the different cell types, in the membranes of the different organelles and in the two halves of each membrane.

216
Q
Which of the following has NOT been observed in prokaryotic cells?
A)	Linear chromosomes
B)	Nucleus
C)	Cytoskeleton
D)	More than one chromosome
E)	All of the above
A

Nucleus

217
Q
Which of the following is a method one could use to isolate and purify the mitochondria of cells away from other cell components?
A)	Flow cytometry and FACS analysis
B)	2D gel electrophoresis
C)	Western blot
D)	Density-gradient centrifugation
E)	Transmission electron microscopy
A

Density-gradient centrifugation

218
Q

List three features that are found in model organisms that are frequently used for research.

A

Features could include: easy to grow in lab, good genetics (easy to isolate mutant, do crosses), genome sequenced, easy to observe development of embryo in microscope, can manipulate genetically (knock-out genes, crosses), possible to reintroduce genes into organisms or generate transgenic individuals.

219
Q

Describe a method one could use to determine where one particular protein is located within a cell.

A

Possible answers include:

a. Immunofluorescent microscopy that uses an antibody against the protein labeled with a fluorescent dye
b. Electron microscopy with antibodies labeled with a heavy metal such as gold
c. Express GFP-tagged version of protein in cell and see where it is located with a fluorescent microscopy
d. Isolate cell fractions and then do western blot with antibody against protein

220
Q

You have isolated a cellular fraction that contains the plasma membrane from mouse liver cells? List all the different types of molecules you will find in this fraction? (possible answers include monosaccharides, polysaccharides, DNA, RNA, nucleotides, proteins, glycoproteins, glycolipids, phospholipids, sphingolipids, steroids, triacylglycerides)

A

Phospholipids, proteins, glycoproteins, glycolipids, steroids (cholesterol), sphingolipids

221
Q

Describe the steps involved in cloning a gene from the human chromosome by using a plasmid vector including how to generate millions of identical copies of the plasmid?

A
  1. Cut the human genomic DNA and the plasmid with the same restriction enzyme
  2. Ligate human genomic fragment into the cut plasmid DNA using DNA ligase
  3. Introduce plasmid DNA into E. coli cells
  4. Grow millions of E. coli cells with the plasmid and then isolate the plasmid DNA from the cells
222
Q

One approach to testing whether keratin intermediate filaments are dynamic is to inject biotin-labeled keratin into living fibroblasts and then to compare, at different times, the distribution of biotin-labeled keratin with that of endogenous keratin intermediate filaments. Assuming that keratin intermediate filaments turn over dynamically with a half-time of one hour, predict the comparative distribution of biotin-labeled keratin and keratin intermediate filaments 10 minutes and four hours after microinjection.

A

Any incorporation of the biotin-labeled keratin into keratin intermediate filaments takes time. After 10 minutes, the injected biotin-labeled keratin should be diffusely distributed in the cell. After four hours, individual keratin intermediate filaments will have turned over (with a one-hour half-time). The biotin-labeled keratin should now be incorporated into the keratin intermediate filaments, and the distribution of the biotin-labeled keratin and of endogenous keratin in the intermediate filaments should be the same.

223
Q

How does Ca2+ molecularly regulate the activity of myosin motors?

A

In striated muscle, actin-myosin contraction is regulated by the binding of Ca2+ to troponin. Troponin binds to actin in the absence of Ca2+ and blocks the binding of myosin to actin. In the presence of Ca2+, troponin changes shape, and myosin can bind to actin. In nonmuscle and smooth muscle cells, myosin activity is regulated by phosphorylation. Myosin light-chain kinase (MLCK) is the responsible enzyme. MLCK activity is regulated by the binding of Ca2+ to calmodulin, which in turn binds to MLCK.

224
Q

Why might colchicine, a tubulin-binding drug, be used to treat cancer?

A

Colchicine, an alkaloid derived from plants, binds tightly to tubulin and inhibits its polymerization, thereby preventing the assembly of the mitotic spindle. The drug acts very quickly and inhibits cell division within a few minutes, hence its usefulness as an anticancer drug.

225
Q

Describe the genetic defect that causes Duchenne’s and Becker’s muscular dystrophy and the molecular processes that characterize the diseases

A

The diseases are caused by a mutation in the gene for dystrophin, a spectrin-related gene that links the actin network beneath the cell surface (the cortex) to transmembrane proteins in the plasma membrane. The transmembrane proteins, in turn, are bound to components of the extracellular matrix, and thus dystrophin plays a role in linking the cortex to the extracellular matrix. This firm anchoring to the extracellular matrix stabilizes muscle cells; without it, the constant stress of contraction results in their destruction and consequently in the loss of muscle tissue that characterizes the diseases.

226
Q

What is one way in which the more rapid growth of actin filaments at one end of the cell (the plus end) compared to the other end (the minus end) is advantageous to the cell?

A

ince the plus end grows 5 to 10 times more rapidly than the minus end, microfilaments essentially grow in one direction. Thus, correct orientation of the plus end will cause a cell to move toward an attractant without any counteracting force in the opposite direction.

227
Q

Actin filaments are a major element of the cytoskeleton. The cytoskeleton provides a framework for the cell and acts as a scaffold that both determines cell shape and positions organelles within cells. For example, the cytoskeleton provides the tracks along which organelles move. At the same time, the cytoskeleton is subject to cleavage by proteins like ADF/cofilin. What is the apparent function of ADF/cofilin?

A

ADF/cofilin is an actin filament; it severs protein that binds to actin freed during filament severing. Actin filaments must be dynamic for the cell to able to move. Hence, the ability of ADF/cofilin to sever actin filaments creates a dynamic actin cytoskeleton.

228
Q

The dynactin protein complex is a regulatory subunit for the minus-end-directed motor protein dynein. Disruption of dynactin will inhibit the activity of cytoplasmic dynein without affecting the activity of kinesin motors. The positioning of various organelles—for example, the Golgi apparatus in cells—is the outcome of a balance between dynein and kinesin. What is the expected distribution of the Golgi apparatus in cells in which the dynactin complex has been disrupted?

A

Disruption of dynactin inhibits dynein activity. This means that there is no minus-end-directed motor tugging at the Golgi apparatus. Kinesin, a plus-end-directed motor, continues to be active, and the Golgi apparatus (or fragments thereof) will be pulled toward the plus end of microtubules (i.e., the cell periphery).

229
Q

In vitro, at tubulin concentrations intermediate between the critical concentration for assembly at the plus and minus ends, microtubules treadmill. As a consequence of treadmilling, the microtubules move. In what direction do the microtubules move and why?

A

Tubulin addition is faster at the plus end than at the minus end of microtubules. There will be an intermediate concentration of free tubulin dimer at which there is net growth at the plus end and net loss of tubulin at the minus end—this is treadmilling. As a result of treadmilling, individual microtubules move in the direction of their plus ends.

230
Q

The kinetic properties of myosin have been optimized such that it moves very rapidly along actin filaments. Specifically, this movement is accomplished by myosin’s very brief attachments to the microfilament along which it moves. In contrast, kinesin moves along microtubules more slowly, with longer periods of attachment between each “step.” How might these differences be explained in evolutionary terms?

A

Because myosin works in conjunction with so many other myosin molecules in muscle contraction, it is unlikely that hundreds of myosin molecules will all let go at once. Therefore, myosin can afford to sacrifice attachment for speed. In contrast, because vesicles and organelles are moved along a microtubule by just a few molecules of kinesin, the chances that they will all let go at once are higher. Therefore, kinesin takes shorter “steps” along the microtubule with longer attachment periods. This increases the chances that the cargo will make it to its final destination without falling off the microtubule.

231
Q

A human hereditary disease has two seemingly unrelated symptoms: an inability to clear mucus from the respiratory system and male sterility. What could be the cause of this disease?

A

The failure of cilia to beat would disable the clearing of the respiratory tract, and the failure of flagella to beat would render sperm immobile—one source of male sterility. This disease is caused by a failure to produce dynein, the motor protein that drives ciliary and flagellar motion. The basis of a third symptom, the occasional placement of the heart to the right side of the thorax instead of to the left, is still unknown

232
Q

Why is Rb, a product of the retinoblastoma gene, now termed a member of a family of proteins designated tumor suppressor genes?

A

Normally, Rb exists in a nonphosphorylated state and is found in complex with the E2F transcription factors that activate genes associated with the cell cycle. When bound by Rb, E2Fs are unavailable for transcription. During initiation of the cell cycle, Cdk4,6/cyclin D phosphorylates Rb, leading to a dissociation of Rb from E2F, leaving the now free E2F available to activate transcription of cell cycle genes. When mutations in Rb prevent its interaction with E2F, E2F is left unregulated; it is available to drive the cell cycle forward and, in essence, induce unrestricted growth or tumor formation. Thus, Rb was designated a tumor suppressor, because in its natural state it can be said to have an intrinsic ability to suppress tumor formation.

233
Q

There are many critical checkpoints in the cell cycle. One in particular ensures that the DNA content is replicated only once. What is the molecular mechanism regulating this process?

A

The key molecule in this restriction of replication is MCM helicase. MCM binds DNA at origins of replication together with the other proteins that make up the origin recognition complex (ORC). The MCMs, in effect, are “licensing factors” which, along with the ORC, are absolutely required for the initiation of replication. The key is that MCM can bind DNA only during G1, and when replication is initiated, MCM is displaced from the origin of replication on the DNA. Therefore, as the cell cycle progresses through S, G2, and M, new rounds of replication cannot be initiated because MCM can bind DNA only in G1.

234
Q

During G2 the Wee1 protein phosphorylates tyrosine-15 of the Cdc2 protein kinase, thus inactivating it. Only once dephosphorylation occurs and Cdc2 becomes active can the cell pass from G2 into M and proceed to cell division. Can you think of a reason why the Wee1 protein was named as such?

A

G2 is a period in the cell cycle during which cell growth occurs. In Wee1 mutant cells, Cdc2 does not become phosphorylated at tyrosine-15 and thus does not acquire the inactivating phosphate. As a result, the cells enter M phase prematurely, before they have time to grow to their full size: hence the name, “wee” mutants.

235
Q

Describe the pathway by which growth factors signal passage through the restriction point in late G1.

A

Extracellular growth factors stimulate the Ras/Raf/ERK pathway, which results in increased synthesis of the D-type cyclins. These then complex with Cdk4, 6, activating their kinase activity. Cdk4, 6/cyclin D then phosphorylates Rb, causing it to dissociate from the E2F transcription factor and allowing E2F to activate its target genes, the products of which are necessary for progression through the restriction point.

236
Q

Single-stranded DNA and unreplicated DNA are recognized by ATR, which leads to G2 arrest. What would likely occur with a single-stranded DNA break in a cell with a mutation that prevented phosphorylation of Chk1? What if the mutation prevented phosphorylation of Chk2?

A

he likely event would be that cells with the Chk1 mutant would not arrest at G2 like their normal counterparts. Once a break is recognized, ATR phosphorylates and activates Chk1, which in turn phosphorylates and inactivates Cdc25. In normal replication, activated Cdc25 activates Cdk1 and the cell cycle progression continues. With a mutation that prevents activation of Chk1, Cdc25 would not be inactivated, Cdk1 would not be blocked, and the cell cycle would continue, allowing any single-stranded damage or unreplicated DNA to be carried forward into daughter cells. A mutation in Chk 2 would not affect single-stranded breaks, because Chk2 is activated by ATM in response to double-stranded breaks. Thus, in response to a single-stranded break, a cell harboring a mutation in the Chk2 would be unaffected and undergo G2 arrest as expected because the ATR–Chk1–Cdc25–Cdk1–G2 arrest pathway would be intact.

237
Q

Normally, following division, cells grow so that daughter cells are relatively comparable in size to the parental cell prior to starting a new cell cycle. Embryonic cells, however, continually get smaller for some time following fertilization. Why is this?

A

Early after fertilization, embryonic cells can complete a round of the cell cycle in as little as 30 minutes. This is accomplished in the absence of growth; thus each division effectively halves the size of the parental cells so that an eight- or sixteen-cell embryo is about the same size as the zygote from which it started. During this time these cells cycle between S and M only, and essentially skip over G1 and G2, the stages at which growth usually occurs.

238
Q

Many cancer researchers and tumor biologists intensely study regulation of cell cycle. Why?

A

If the cell cycle begins to run unchecked—if normal regulation fails—the affected cell and its progeny divide out of control and become a tumor that may, in turn, become malignant.

239
Q

Describe important ways in which meiosis differs from mitosis.

A

Mitosis produces two daughter nuclei genetically identical to each other and to the nucleus that underwent division; meiosis produces four daughter nuclei genetically different from one another and from the original nucleus, and each daughter nucleus has half as many chromosomes as did the original nucleus. Mitosis is a single nuclear division; meiosis consists of two nuclear divisions. Mitosis is a device for constancy; meiosis is a device for diversity. (There are other differences as well.)

240
Q

One of the hallmark characteristics of apoptosis is fragmentation of nuclear DNA. Which protein is responsible for this fragmentation, and what is the mechanism by which it degrades the DNA?

A

Caspases are the apoptotic enzymes responsible for fragmentation of nuclear DNA. All cells have DNase enzymes that digest and fragment nuclear DNA. Under normal conditions various proteins function as inhibitors of DNase. In apoptosis, caspases block the DNase inhibitor, which in effect activates the DNase and allows for the commencement of the fragmentation process. This happens early in apoptosis and can be observed in 1–3 hours.

241
Q

Bcl-2 is an antiapoptotic protein which, under normal conditions, contributes to the balance of activity of numerous proapoptotic and antiapoptotic proteins. What would be the result of a gain-of-function mutation in the bcl-2 gene that led to a constant and significant elevation in cellular Bcl-2 protein levels?

A

A gain-of-function mutation can, in fact, occur in the bcl-2 gene, and its discovery led to the identification of the oncogene bcl-2 in some B-cell lymphoma cancers. If Bcl-2 becomes overexpressed, the balance is tipped in favor of Bcl-2 and apoptosis is blocked, even under conditions that normally would induce cell death. The effect is proliferation of unhealthy cells that normally would be eliminated by apoptosis.

242
Q

Mammalian cells are constantly dying, whether they are undergoing apoptosis or die by acute injury. What is the key difference between the two processes?

A

Apoptotic cells and cell fragments are recognized and efficiently removed from surrounding tissues via phagocytotic macrophages with no pathophysiological consequence. It is an active and programmed process. Cells subjected to acute injury swell, lyse, and spill into the extracellular space. This release leads to tissue inflammation. If apoptosis had similar consequences, the subsequent inflammation likely would not allow an organism to survive.

243
Q

Signaling pathways mediate many cellular processes, as explained in Chapter 15. Apoptosis is also regulated by multiple signaling pathways. Briefly describe two signaling pathways that regulate apoptosis

A

One regulator of apoptosis is the p53 pathway. Upon DNA damage, the ATM and Chk2 protein kinases phosphorylate and stabilize p53, which in turn transcribes the genes encoding PUMA and Noxa. PUMA and Noxa are proapoptotic BH3-only proteins that initiate apoptosis. Another is the PI 3-kinase pathway, which can be activated by either protein tyrosine kinases or G protein-coupled receptors. Once activated, PI 3-kinase phosphorylates PIP2 to PIP3; this activates Akt, which in turn phosphorylates another BH-3-only proapoptotic protein, Bad. Another pathway involves activation of caspase 9 by TNF-alpha receptors.

244
Q

Briefly describe the steps involved in cloning using somatic cell nuclear transfer. Give an example.

A

The nucleus of an adult somatic cell (nongonadal cell, such as an epithelial cell) is transferred to an unfertilized egg that has been enucleated (meaning the nucleus of the egg has been removed). The egg is then cultured in vitro to an early blastocyst embryo and implanted into a foster mother who was not the source of the somatic cell nucleus. The foster mother gives birth to a clone that is identical to the donor of the somatic cell. By far the most well-known example is Dolly the sheep, which was cloned by Ian Wilmut’s group in 1997. Since then, numerous species have been cloned.

245
Q

When fragmented nuclear DNA is visualized by gel electrophoresis following induction of apoptosis, why do the fragments almost always appear as multiples of 200 base pairs?

A

DNA exists in the nucleus as chromatin and it is intimately associated with many proteins. The basic structural unit of chromatin is the nucleosome, which contains 200 base pairs of DNA wrapped around a histone core. The linker DNA, which consists of about 50 base pairs, is particularly sensitive to digestion by nucleases such as DNase, but the 200 base pairs within each nucleosome are fairly well protected from digestion. Therefore, when run on a gel that denatures the proteins, the DNA strands that are observed are multiples of 200.

246
Q

What is a possible reason that carcinomas, which are tumors derived from epithelial cells, are the most common types of cancer in humans?

A

Most cells in adults have ceased to proliferate and enter the G0, or quiescent, stage of the cell cycle. In contrast, epithelial cells are continually turned over and thus continue to divide. This may make them more susceptible to uncontrolled cell proliferation, because many of the signals that stimulate cell division are still active. In addition, epithelial cells line the inner and outer surfaces of the body, such as the intestinal and respiratory tracts, and the skin. Thus they come into frequent contact with cancer-causing substances in the environment.

247
Q

Why is cancer more often cured when detected early?

A

Cancer develops in a stepwise fashion: a cell first begins to proliferate uncontrollably. This is followed by an increased invasiveness of one of the cell’s progeny, followed by an increased metastatic potential of one of that cell’s progeny. Thus, the gradual accumulation of mutations leads to a malignant tumor with metastatic potential. If the tumor is caught early on, at the stage where it is still just a benign growth, it can be treated locally by surgery. In addition, the earlier a malignant tumor is treated, the more likely it will be to respond to radiation and chemotherapy; unlike normal cells, cells in late-stage tumors often do not respond to these treatments by ceasing cell proliferation.

248
Q

What is the therapeutic strategy for using Herceptin to treat breast cancer?

A

In about 30% of breast cancers, ErbB-2 (a receptor protein tyrosine kinase) is overexpressed as a result of amplification of the erbB-2 gene. In vitro, cells that overexpress ErbB-2 exhibit elevated proliferation. Herceptin is a monoclonal antibody that binds the extracellular domain of ErbB-2 and blocks proliferation in the ErbB-2 overexpressing cell lines. In clinical trials it was shown to reduce tumor size and increase human survival, and it became the first antibody to gain FDA approval. It is now widely used in treating ErbB-2-overexpressing breast tumors.

249
Q

Why are cancer cells less dependent on growth factors than their normal cellular counterparts?

A

One reason is that cancer cells frequently secrete their own growth factors that act in an autocrine fashion, making them less dependent on growth factors from normal signaling pathways. Another is that frequently, oncogenic mutations occur in the signaling pathways themselves, making the pathways constitutively active and less dependent on any growth factors that would normally trigger the pathways (in effect they bypass the initiating event of the cascade altogether).

250
Q

One of the first viruses to be linked to cancer was the Epstein-Barr virus. What is the mechanism by which it is thought to transform cells?

A

The major transforming protein encoded by the Epstein-Barr virus genome is LMP-1, which mimics a cell surface receptor on B lymphocytes. This faux receptor activates signaling pathways to stimulate cell division and inhibit apoptosis, leading to overgrowth of cells with no apoptotic destruction.

251
Q

What is the likely reason that antiangiogenic therapies hold so much promise for treatment of tumors?

A

Without angiogenesis, tumors would be limited in size to approximately a million cells. After that, angiogenesis is required for the tumor to have adequate blood flow for survival. Blood vessels grow in response to growth factors secreted by the tumors, which stimulate the endothelial cells to proliferate in the walls of capillaries in surrounding tissues. This results in the outgrowth of new capillaries into the tumor and also facilitates metastasis. Thus, drugs that block this angiogenesis could have a dramatic effect on tumor growth and metastasis. These drugs are currently in clinical trials and development in many pharmaceutical companies and laboratories.

252
Q

Cellular transformation from normal to tumor cells is a hallmark of cancer. The focus assay represents the first and most widely used assay to assess cell transformation. Describe the focus assay.

A

The focus assay is based on the ability to recognize a group of transformed cells as a morphologically distinct “focus” against a background of normal cells on the surface of a culture dish. The focus assay takes advantage of three properties of transformed cells: altered morphology, loss of contact inhibition, and loss of density-dependent inhibition of growth. The result is the formation of a colony of morphologically altered cells that overgrow the background of normal cells. These foci of transformed cells can be detected from within a few days to two weeks.